diseño instruccional matemáticas. “trigonometría y...

98
Diseño Instruccional Matemáticas. “Trigonometría y Geometría analíticaProyecto: Diseño e implementación de un programa para la actualización de docentes de nivel medio superior en las áreas de físico-matemáticas y ciencias naturales. Responsable Técnico: Silvia Melbi Gaona Jiménez. Supervisión y revisión del documento: Silvia Melbi Gaona Jiménez Contenido Temario ............................................................................................................................................... 2 Banco de preguntas........................................................................................................................... 62 Ejercicios............................................................................................................................................ 84 Vectores ........................................................................................................................................ 84 Puntos y rectas .............................................................................................................................. 87 Lugares geométricos I ................................................................................................................... 88 Lugares geométricos II .................................................................................................................. 89 Coordenadas Polares..................................................................................................................... 93 Ejercicios de Aplicación ................................................................................................................. 95 Tangentes a cónicas ...................................................................................................................... 97

Upload: others

Post on 16-Mar-2020

8 views

Category:

Documents


0 download

TRANSCRIPT

Page 1: Diseño Instruccional Matemáticas. “Trigonometría y ...conacyt.upemor.edu.mx/informe3/archivos/PropiedadInt...Diseño Instruccional Matemáticas.“Trigonometría y Geometría

Diseño Instruccional

Matemáticas. “Trigonometría y Geometría analítica”

Proyecto: Diseño e implementación de un programa para la actualización de

docentes de nivel medio superior en las áreas de físico-matemáticas y

ciencias naturales.

Responsable Técnico: Silvia Melbi Gaona Jiménez.

Supervisión y revisión del documento: Silvia Melbi Gaona Jiménez

Contenido

Temario ............................................................................................................................................... 2

Banco de preguntas........................................................................................................................... 62

Ejercicios ............................................................................................................................................ 84

Vectores ........................................................................................................................................ 84

Puntos y rectas .............................................................................................................................. 87

Lugares geométricos I ................................................................................................................... 88

Lugares geométricos II .................................................................................................................. 89

Coordenadas Polares..................................................................................................................... 93

Ejercicios de Aplicación ................................................................................................................. 95

Tangentes a cónicas ...................................................................................................................... 97

Page 2: Diseño Instruccional Matemáticas. “Trigonometría y ...conacyt.upemor.edu.mx/informe3/archivos/PropiedadInt...Diseño Instruccional Matemáticas.“Trigonometría y Geometría

2

Temario

Examen Diagnostico

Módulo I. Introducción

¡Bienvenidos a este curso de geometría analítica!

Éste no es un curso convencional de geometría analítica porque es un curso en línea o un curso

"virtual", como se le quiera llamar. Pero lo más importante es que en este curso se busca sacar

ventaja de que quien lo tome está conectado a Internet y por tanto tiene en sus manos la fuente

de información y autoaprendizaje más importante que ha creado el hombre. Entonces, su objetivo

principal no es enseñar la geometría analítica a personas que ya enseñan geometría analítica, sino

más bien, tomar como pretexto el enseñar cómo se hace geometría analítica usando vectores (un

método poco usado en las aulas), para mostrar una manera de incorporar la tecnología

informática en la práctica docente.

Es importante empezar el curso con una explicación de su estructura y funcionamiento. Se puede

decir que este curso virtual está diseñado en base a lecciones y evaluaciones semanales donde se

encuentran integrados contenidos explicativos sobre los temas y ejercicios de práctica

relacionados. En este caso una lección es una presentación estructurada de un tema con

contenidos y ejercicios. Dentro de los contenidos se encuentran las definiciones y explicaciones

necesarias junto con enlaces a documentos externos en conceptos claves. Estos enlaces se

encuentran como se muestra aquí en forma de hipervínculos. Al hacer "click" se despliegan y

permiten hacer revisiones más extensas del concepto en cuestión. Muchos de estos enlaces son a

la enciclopedia conocida como Wikipedia, que es un proyecto educativo muy importante a nivel

mundial , pues genera una enorme cantidad de información con un buen estándar de calidad. Los

artículos con mala estructura o información dudosa siempre presentan advertencias. Toda la

información contenida es de libre acceso y por eso es especialmente importante. Su intención es

transmitir cultura a la mayor cantidad de personas en el planeta, por eso es necesario que tanto

profesores como alumnos la conozcan y aprovechen.

También, dentro del contenido se encontrarán múltiples construcciones geométricas, útiles en un

curso como éste. Estas construcciones están diseñadas en una aplicación llamada "Geogebra"

(www.geogebra.org), otro programa de libre uso y distribución para fines no comerciales. El

Geogebra es una poderosa herramienta para la enseñanza de las matemáticas, puede usarse en

cualquier computadora personal y con cualquier sistema operativo. De hecho los invitamos a

instalarlo (http://www.geogebra.org/cms/es/installers) en sus propias computadoras personales

para usarlo durante este curso y como material en sus propios cursos.

El curso no requiere la instalación de geogebra, pues las lecciones son autocontenidas. Pero los

exhortamos a instalarlo y practicar su uso, la aplicación se puede considerar amable e intuitiva

Page 3: Diseño Instruccional Matemáticas. “Trigonometría y ...conacyt.upemor.edu.mx/informe3/archivos/PropiedadInt...Diseño Instruccional Matemáticas.“Trigonometría y Geometría

3

pero si es necesario, el manual también puede encontrarse en línea. No vamos a hacer un curso de

geogebra aquí, pero sí es necesario presentar algunos puntos generales de su operación.

En la siguiente figura se observa una ventana de geogebra:

Podemos distinguir las partes importantes de la aplicación, que son: una barra de herramientas y

una barra de menú donde se encuentran todas las configuraciones y acciones a que se tiene

acceso como usuario; el área principal está dividida en dos partes, una que es el área gráfica

donde los dibujos aparecen y una ventana algebraica (panel derecho) donde se listan todos los

objetos de la construcción, pero en su definición abstracta, es decir, aparecen las ecuaciones y

coordenadas de los objetos dibujados. En la parte baja de la ventana hay una barra de entrada y

una lista de comandos, ahí se pueden capturar ecuaciones o funciones contenidas en la lista, que

sirven para hacer cálculos avanzados.

Noten algo importante, la figura que se muestra es interactiva, de hecho es el programa completo

ejecutándose dentro de este documento. Prueben a usarlo, con los botones en las barras se

pueden añadir puntos, vectores, líneas, curvas cónicas y muchas cosas más. Como primera

actividad practiquen a usarlo aquí mismo. También pueden capturar ecuaciones, por ejemplo en la

línea de entrada prueben teclear: "x^2+y^2=3" y chequen el dibujo.

Page 4: Diseño Instruccional Matemáticas. “Trigonometría y ...conacyt.upemor.edu.mx/informe3/archivos/PropiedadInt...Diseño Instruccional Matemáticas.“Trigonometría y Geometría

4

En las lecciones encontrarán ventanas como ésta, pero mostrando sólo los componentes que

nosotros seleccionemos para la realizar la actividad en cuestión. Si se trata de una animación o

construcción automática busquen los botones de inicio dentro de la ventana gráfica que

usualmente muestran un triángulo. Debido a la interactividad de las construcciones este curso

está pensado para usarse de equipos conectados a internet con bandas anchas. Si la conexión es

lenta, la carga de las construcciones puede tomar tiempo, si éste es su caso, es necesario tener un

poco de paciencia.

La lección correspondiente a cada módulo es la actividad principal de éste . Se puede navegar en

ella cuantas veces se quiera, pero para hacer la evaluación correspondiente, es necesario

terminarla al menos una vez. Los ejercicios dentro de las lecciones se evalúan sólo como

mecanismo de retroalimentación. Si alguien decide terminar la lección sin realizar los ejercicios, no

tendrá ninguna retroalimentación. La calificación del módulo depende sólo del resultado del

examen correspondiente.

Esperamos que disfruten el curso, como nosotros hemos disfrutado su elaboración.

1. Vectores

1.1. Definición de un vector y sus propiedades

Una buena manera de definir lo que es un vector es mediante la noción que

intuitivamente tenemos de una flecha. Es decir es una figura geométrica que

normalmente nos indica una dirección, pero que además tiene las siguientes propiedades:

1. Tiene un tamaño

2. Indica una dirección

3. Tiene un punto del cual comienza

Matemáticamente, los vectores son el conjunto de todas las flechas que podamos

imaginar. Así podemos pensar en flechas sobre una recta, en el plano, en el espacio y

hasta en hiperespacios. Aquí nos concentraremos en los que viven en un plano, ya que la

geometría analítica plana puede sacar mucha ventaja de estos.

En la siguiente figura tenemos un ejemplo interactivo que incluye algunos vectores en un

plano cartesiano. En este ejemplo es posible, usando el ratón, desplazar los vectores

libremente, sin que pierdan su forma (simplemente haz click con el botón izquierdo sobre

el centro del vector y mantenlo oprimido mientras lo trasladas). También es posible añadir

nuevos vectores utilizando la barra de entrada; por ejemplo tecleando el comando:

vector[(2,3),(1,-1)], se dibujará un vector que va del punto (2,3) al punto (1,-1).

Page 5: Diseño Instruccional Matemáticas. “Trigonometría y ...conacyt.upemor.edu.mx/informe3/archivos/PropiedadInt...Diseño Instruccional Matemáticas.“Trigonometría y Geometría

5

Por esta propiedad se dice que los vectores son ``libres'' porque no importa donde se

coloquen siempre son iguales. Una vez colocados en un plano cartesiano un vector se

asocia inmediatamente con dos puntos del plano (por ejemplo el vector z se encuentra

asociado a los puntos G y H). Si cambio uno de los puntos el vector cambia su forma (se

puede probar en el ejemplo a hacerlo). Así se establece una especie de función entre

puntos y vectores. Para que esta función sea biyectiva, es decir que, haya una relación

única entre un punto y un vector es necesario trasladar a todos nuestro vectores a un

punto común, que podría ser al mismo tiempo el origen del plano cartesiano. El ejemplo

siguiente los dibuja así:

En el ejemplo anterior también es posible añadir más vectores centrados, tecleando en la

barra de entrada una letra minúscula y una coordenada de la siguiente manera: f=(4,2);

este comando que se dibujará el vector f del origen al punto (4,2).

Ahora se puede notar que los vectores perdieron libertad por estar atados al centro, pero

ganamos la posibilidad de matematizarlos, porque un vector ahora puede ser

representado por un punto en el plano y por tanto, usando ejes cartesianos, convertirlo en

un par de números ordenados (x,y). La importancia de esto radica en que ahora podemos

medir sus propiedades y definir operaciones entre ellos. Así para un vector u asociado a la

pareja (x,y) su tamaño se puede encontrar usando el \vert u\vert es el tamaño del vector

u, entonces

\begin{displaymath} \vert u\vert=\sqrt{x^2+y^2}. \end{displaymath}

Page 6: Diseño Instruccional Matemáticas. “Trigonometría y ...conacyt.upemor.edu.mx/informe3/archivos/PropiedadInt...Diseño Instruccional Matemáticas.“Trigonometría y Geometría

6

(1)

Si quisiéramos saber el ángulo e inclinación del vector u, representado por \theta_u

entonces se tendría que echar mano de la trigonometría, usando la función tangente de

un triángulo rectángulo, de modo que es sencillo demostrar que:

\begin{displaymath} \theta_u=\arctan \frac{y}{x}. \end{displaymath}

(2)

Todo vector, esté donde esté, debe trasladarse al centro de los ejes cartesianos para

poderlo medir sus propiedades con las dos definiciones anteriores.

En la siguiente construcción se puede observar el tamaño y el ángulo de inclinación de un

vector, si se hace click en el punto B y se deja presionado el botón es posible cambiar el

vector u y sus propiedades se recalculan automáticamente. Es de notar que los vectores v

y w son vectores que apuntan en las direcciones de los ejes cartesianos sus tamaños

coinciden con las coordenadas x y y de u respectivamente formando siempre triángulos

rectángulos y por tanto relacionados con el teorema de Pitágoras y las funciones

trigonométricas.

1.2. Suma de vectores

Además de sus propiedades, también se pueden definir operaciones entre vectores. Así,

podemos definir la suma de vectores como: si tenemos dos vectores v y w descritos por

los puntos A=(x_1,y_1) y B=(x_2,y_2), la suma será:

v+w=(x_1+x_2,y_1+y_2).

(3)

En palabras, la suma de dos vectores es otro vector, tal que su abscisa es la suma de las

dos abscisas originales y su ordenada, la suma de las dos ordenadas. Al aplicar esta

definición y dibujarla, nos encontramos que se forma una figura como la siguiente:

Page 7: Diseño Instruccional Matemáticas. “Trigonometría y ...conacyt.upemor.edu.mx/informe3/archivos/PropiedadInt...Diseño Instruccional Matemáticas.“Trigonometría y Geometría

7

Nótese que la suma de los vectores u y v es w y que el punto D donde llega el vector w es

siempre la suma por competentes de los puntos B y C (que a su vez representan las puntas

de los vectores u y v).

Si un vector tiene coordenadas (x,y) el vector (-x,-y), que le podríamos llamar su negativo,

es un flecha que apunta en la misma dirección, pero en sentido contrario. La resta de dos

vectores se puede definir por la suma de un vector por el negativo de otro. Es decir:

u-v=u+(-v)=(x_1 - x_2, y_1 - y_2 ) .

El vector que resulta de la resta de u y v, coincide siempre en tamaño y dirección con un

vector que va del punto (x_2,y_2) al punto (x_1,y_1).

Para terminar, un comentario sobre la suma y la resta en el Geogebra. Una vez capturado

un vector el Geogebra puede hacer las operaciones vectoriales usando la barra de entrada

de comandos. De esta manera, tenemos dos vectores capturados en u y v el comando:

u+v, calcula y dibuja el valor de la suma y el comando: u-v, la resta.

Hasta aquí con las definiciones asociadas a la suma, el flujo de la lección aquí permite las

siguientes opciones: Seguir con ejercicios de esta sección o continuar revisando

contenidos. La recomendación es hacer los ejercicios primero.

1.3. Productos vectoriales

La segunda operación importante es el producto por escalar. Si tenemos un vector

v=(x_1,y_1) y un escalar t el producto entre ambos se define como:

t v=(t x_1, t x_2).

Este producto tiene la característica de no alterar la dirección del vector y sólo cambiar su

tamaño. Se puede ver en el ejemplo siguiente esta característica, prueba la animación.

Page 8: Diseño Instruccional Matemáticas. “Trigonometría y ...conacyt.upemor.edu.mx/informe3/archivos/PropiedadInt...Diseño Instruccional Matemáticas.“Trigonometría y Geometría

8

La última operación que definiremos es el producto escalar entre dos vectores. Cuando se

tienen dos vectores en el plano dados por v=(x_1,y_1) y u=(x_2,y_2) su producto escalar se

define como:

v \cdot u=x_1 x_2+y_1 y_2.

La primera implicación de esta definición es que si se toma el producto de un vector

u=(x,y) consigo mismo, es decir u \cdot u , el resultado coincide con la definición de su

tamaño, pero al cuadrado, es decir:

u \cdot u = x^2 + y^2 = |u|^2.

Ahora, observa la siguiente construcción, es un poco más elaborada, pero se explicará con

más detalle. En ella se puede notar que hay cuatro vectores u, v, w y z. El ángulo \alpha es

el ángulo entre u y v o entre w y z. El producto escalar de u y v se calcula automáticamente

y para comparar también se evalúa y se imprime la cantidad |u||v| \cos \alpha. Es

importante observar que ambas cantidades son iguales y si se cambia el tamaño de u o v

haciendo "click" y arrastrando cualquiera de sus puntas, siempre se conserva esa igualdad.

Por tanto, la construcción demuestra que la igualdad siguiente se cumple siempre:

u \cdot v = |u||v| \cos \alpha

También es importante remarcar que si el ángulo de inclinación entre ellos es un ángulo

recto entonces el producto punto es cero. En otras palabras, vectores perpendiculares

siempre tienen producto punto nulo. Es posible verificarlo en la misma construcción, de

modo que al mover el punto B o C hasta que los vectores sean perpendiculares sus

productos se hacen cero. Si los vectores fueran paralelos entonces su producto punto

sería igual al producto de sus magnitudes.

El segundo concepto importante que puede obtenerse de la figura anterior está

relacionado con los vectores w y z. El vector w se observa como una proyección del vector

v sobre el vector u. Además se puede observar que sin importar como se cambie u o v

siempre los tamaños de w y z son:

|w|=\frac{v \cdot u}{|u|},

Page 9: Diseño Instruccional Matemáticas. “Trigonometría y ...conacyt.upemor.edu.mx/informe3/archivos/PropiedadInt...Diseño Instruccional Matemáticas.“Trigonometría y Geometría

9

|z|=\frac{v \cdot u}{|v|}.

Es decir, el tamaño de la proyección del vector v sobre el vector u es siempre el producto

punto de ambos entre el tamaño de u y viceversa, la proyección de u sobre v es el

producto puto de ambos entre v.

Dentro de una ventana de geogebra también es posible calcular los productos vectoriales.

Si se capturan un escalar t y dos vectores u y v, es posible hacer productos el producto por

escalar, por ejemplo con el comando: t*u, y el producto vectorial con la sintaxis u*v.

Cualquier producto se calcula con el símbolo "*" y el programa automáticamente

identifica si es un producto por escalar o un producto punto.

Las operaciones de suma, resta y multiplicación son las operaciones básicas entre vectores

y son suficientes para plantear una gran variedad de problemas geométricos.

1.4. Ejercicios

2. Cuestionario

Módulo II. Puntos y Rectas

Un plano cartesiano es un sistema de referencia de un espacio de 2 dimensiones. Un punto es

cualquier ubicación sobre el plano cartesiano. Para describir la ubicación de un punto sobre el

plano cartesiano se utiliza el sistema de referencia del plano, conformado por 2 ejes \left(x,y\right)

y un origen. Los puntos también son conocidos como par ordenado, ya que la forma común de

representarlos es \left(n1,n2\right) donde cada uno de los números representa coordenadas en

cada uno de los ejes del plano.

Para representar un plano se utilizan 2 líneas, una horizontal y otra vertical que se cruzan de forma

perpendicular. Al punto exacto del cruce de los ejes se le llama origen y se representa con el par

ordenado \left(0,0\right). Los ejes forman 4 cuadrantes en los cuales se podrán ubicar puntos.

Page 10: Diseño Instruccional Matemáticas. “Trigonometría y ...conacyt.upemor.edu.mx/informe3/archivos/PropiedadInt...Diseño Instruccional Matemáticas.“Trigonometría y Geometría

10

Relación punto-vector.

Si tenemos un punto A ubicado en (1,1) y lo movemos un punto a la izquierda y un punto arriba

vemos que ahora se ubica en (0,2). A este concepto se llama desplazamiento. Partiendo de este

concepto para representar la translación de un par ordenado en el plano, también podemos

llamar vector a un punto en el plano. Esto se representa con una flecha a la cual llamamos vector

geométrico.

Segmento: Distancia entre dos puntos.

Dado un punto A y un punto B, la recta comprendida entre ellos se llama segmento. Los puntos A y

B se llamaran extremos del segmento y la recta será conocida como segmento AB.

Page 11: Diseño Instruccional Matemáticas. “Trigonometría y ...conacyt.upemor.edu.mx/informe3/archivos/PropiedadInt...Diseño Instruccional Matemáticas.“Trigonometría y Geometría

11

Recta.

En su definición más simple una recta es un conjunto infinito de puntos sobre un plano. Una recta

se puede definir de forma algebraica como

Ax+By+C=0

siempre que A o B sea diferente a 0.

Con base en un plano cartesiano podemos deducir que la unión de 2 vectores se le llama recta,

entiendo por vector a un par ordenado de coordenadas (x,y). Así podemos ver que una recta se

puede obtener mediante una ecuación vectorial.

Si tenemos que A=(4,4) y B=(3,1) son puntos que pasan por la recta a, podemos aplicar la siguiente

fórmula para determinar el vector sobre esa recta:

\nu=A-B=(5,4)-(2,1)=(3,3)

Ejemplo 1. Dados los puntos A=(2,2) y B=(1,-1) obtener el valor del vector AB. Usando la formula

anterior podemos ver que \nu=(1,3). Si a esto agregamos la representación geométrica ordinaria

de los vectores a y b que corresponden a los puntos A y B respectivamente obtenemos:

Ecuación paramétrica vectorial.

La ecuación paramétrica vectorial sirve para obtener puntos que pasen por una recta dada por 2

puntos. La ecuación es:

Page 12: Diseño Instruccional Matemáticas. “Trigonometría y ...conacyt.upemor.edu.mx/informe3/archivos/PropiedadInt...Diseño Instruccional Matemáticas.“Trigonometría y Geometría

12

c = b + r (a-b)

Ejemplo 2. Obtener la ecuación paramétrica vectorial de la recta a que pasa por A=(-1,3) y B=(-5,-

2).

Para iniciar trazamos un diagrama con los puntos AB. Posteriormente obtenemos:

a-b=(-1,3)-(-5,-2)=(4,5)

Sustituyendo en la ecuación paramétrica vectorial obtenemos:

c=(-5,-2)+r(4,5)

Si restringimos el valor de r a un intervalo cerrado {r:a=r=b} entonces lo que obtenemos en la

gráfica es un segmento de recta que recorre desde B hasta A. Si r=0 entonces c=b, si r=1 entonces

c=A. Si queremos un resultado menor a b entonces r<0 y si queremos un resultado mayor a a

entonces r>1.

1. División de segmentos en una razón dada

Una razón es una división o fracción, por ejemplo AC/CB es una razón. Estas razones se usan

para determinar las proporciones de un objeto. ¿Qué significa la división de un segmento en

una razón dada? Significa medir el segmento y partirlo, y las medidas de cada pedazo de

segmento, al dividirlas, tienen que dar igual a la razón que buscamos.

Con estos conceptos partiremos para encontrar las ecuaciones que nos permitan encontrar un

punto sobre el segmento. Analicemos primero el escenario de forma análoga, teniendo

A(x1,y1) y B(x2,y2) y asumiendo que r=AC/CB podemos enfocarnos en obtener x.

NOTA: El orden en el cual nos dan un segmento es importante, así que debemos de poner

especial atención en este punto.

r=\frac{AC}{CB}=\frac{x1-x}{x-x2}

r(x-x2)=x1-x

Page 13: Diseño Instruccional Matemáticas. “Trigonometría y ...conacyt.upemor.edu.mx/informe3/archivos/PropiedadInt...Diseño Instruccional Matemáticas.“Trigonometría y Geometría

13

rx-rx2=x1-x

rx+x=x1+rx2, factorizando obtenemos

x(r+1)=x1+rx2

x=\frac{x1+rx2}{1+r}

El mismo procedimiento lo enfocamos para obtener y:

y=\frac{y1+ry2}{1+r}

Nota: Si r es un valor negativo entendamos que este se encuentra fuera del segmento dado. Si

r es un valor positivo se encuentra dentro del segmento.

2. Pendiente de una recta

Una recta se puede describir considerando un vector w que tiene la misma dirección que la

recta, y un punto de inicio del vector. El caso más sencillo es considerar un vector w , y como

punto inicial el origen. El vector con punto inicial en el origen se puede expresar como w=(h,k).

En la siguiente figura se observa que el vector w forma un ángulo \theta con el eje x. Al utilizar

esta geometría se define la pendiente de una recta L como:

m=Tan\:\theta=\frac{k}{h},

En caso de que h=1, entonces, m=Tan\:\theta=k,

Una recta L que no intercepta al origen, y contiene a los puntos A(x_1,y_1) y B(x_2,y_2), se

puede representar vectorialmente que se muestra en la siguiente figura. Ahí se puede ver que,

del origen C al punto A se tiene al vector u, y del origen C al punto B se tiene al vector v, y por

lo tanto, el vector diferencia es w=v-u, el cual respecto a las coordenadas se puede expresar

Page 14: Diseño Instruccional Matemáticas. “Trigonometría y ...conacyt.upemor.edu.mx/informe3/archivos/PropiedadInt...Diseño Instruccional Matemáticas.“Trigonometría y Geometría

14

como w=(x_2-x_1,y_2-y_1). Utilizando la definición de pendiente aplicada a este caso se tiene

que:

m=Tan\:\theta=\frac{y_2-y_1}{x_2-x_1},

En la figura se representa la pendiente como:

m=Tan\:\theta=\frac{k}{h}=\frac{\frac{y_2-y_1}{x_2-x_1}}{1},

es decir,

h=1, y m=Tan\:\theta=k=\frac{y_2-y_1}{x_2-x_1},

3. Ángulo entre dos rectas

En la siguiente figura se muestran dos rectas L_1 y L_2, que se interceptan en el punto E. La

intercepción forma cuatro vértices, y el vértice superior de la intercepción tiene un ángulo

\theta, que es considerado el ángulo entre las rectas. El ángulo \theta que se forma en la

intersección de las rectas, cumple por la geometría que \theta=\alpha-\beta, por lo tanto, se

tiene que Tan\:\theta=Tan(\alpha-\beta), y al usar la identidad trigonométrica para la

tangente de la diferencia de dos ángulos se tiene que:

Tan\:\theta=Tan(\alpha-\beta)=\frac{Tan\:\alpha-

Tan\:\beta}{1+(Tan\:\alpha)(Tan\:\beta)}=\frac{m_2-m_1}{1+m_1m_2}

En esta relación se ha considerado que, la pendiente de la recta L_2 es m_2=Tan\:\beta, y la

pendiente de la recta L_1 es m_1=Tan\:\alpha.

Page 15: Diseño Instruccional Matemáticas. “Trigonometría y ...conacyt.upemor.edu.mx/informe3/archivos/PropiedadInt...Diseño Instruccional Matemáticas.“Trigonometría y Geometría

15

Por otro lado, también se puede relacionar el ángulo entre dos rectas, al considerar los

vectores v y w, y al utilizar la relación del producto punto con el Cos\:\theta, se obtiene que:

Cos\:\theta=\frac{v\cdot w}{\left\|v\right\|\left\|w\right\|}

4. Rectas paralelas

Sabemos de geometría elemental euclidiana que dos rectas en el plano son paralelas si no se

cortan en ningún punto. Esto quiere decir, las rectas tienen iguales ángulos que cortan el eje x,

es decir, \alpha=\beta. Por lo tanto, \theta=0^{\circ}. Al considerar esto en la relación

Tan\:\theta, se tiene que,

Tan\:\theta=0=\frac{m_2-m_1}{1+m_1m_2}

Es decir, m_1=m_2. Por lo tanto, dos rectas paralelas tienen igual pendiente. También, el

ángulo \theta entre dos rectas paralelas se puede encontrar al considerar la relación del

coseno de \theta, evaluado cuando, \theta=0^{\circ}, o bien, \theta=180^{\circ}. Al considerar

Page 16: Diseño Instruccional Matemáticas. “Trigonometría y ...conacyt.upemor.edu.mx/informe3/archivos/PropiedadInt...Diseño Instruccional Matemáticas.“Trigonometría y Geometría

16

que el vector v es contenido en la recta L_1, y el vector w está contenido en la recta L_2,

entonces, al aplicarse los dos valores al ángulo \theta, se encuentra que,

\frac{v\cdot w}{\left\|v\right\|\left\|w\right\|}=\pm1

El signo "+" corresponde a \theta=0^{\circ}, y el signo "-" corresponde a theta=180^{\circ}.

Ésta es la relación que resume la condición de paralelismo entre dos rectas, cuando se

conocen un vector contenido en cada una de las rectas.

5. Rectas perpendiculares

Dos rectas son perpendiculares si forman éstas un ángulo \theta=90^\circ al interceptarse.

Para obtener la condición de las pendientes de dos rectas perpendiculares, se considerará la

función inversa de la reciproca de la tangente, es decir la cotangente. Sabemos que,

Cot\:\theta=\frac{1}{Tan\:\theta},

Al aplicar la relación de Tan\:\theta con respecto a las pendientes de las rectas se obtiene:

Cot\:\theta=\frac{1+m_1m_2}{m_2-m_1}

Al aplicar en la función cotangente, \theta=90^\circ, se tiene que,

Cot\:90^\circ=0=\frac{1+m_1m_2}{m_2-m_1}

Por lo tanto, cuando dos rectas son perpendiculares sus pendientes cumplen la relación,

m_1m_2=-1.

Al considerar que el vector v es contenido en la recta L_1, y el vector w está contenido en la

recta L_2, entonces, al aplicarse que \theta=90^\circ en la relación del Cos\:\theta, y se

encuentra que,

Cos\:90^\circ=0=\frac{v\cdot w}{\left\|v\right\|\left\|w\right\|}

Es decir, v\cdot w=0, que es la condición de perpendicularidad de dos rectas, o bien, de los dos

vectores contenidos en cada una de las rectas.

6. Ecuaciones de la recta

La recta puede describirse analíticamente con diferentes expresiones, se iniciará con la

descripción paramétrica de ésta.

Page 17: Diseño Instruccional Matemáticas. “Trigonometría y ...conacyt.upemor.edu.mx/informe3/archivos/PropiedadInt...Diseño Instruccional Matemáticas.“Trigonometría y Geometría

17

Para describir la recta en forma paramétrica se considera que la recta contiene al vector

u=(h,k), y que este vector inicia en el punto P_0 (x_0,y_0). Cualquier punto en la recta se

representa como P(x,y). Con este punto se obtiene el vector

\stackrel{\rightarrow}{PP_0}=v=(x-x_0,y-y_0), y dado que los vectores u y v pertenecen a la

recta, entonces son paralelos, por lo tanto, deben cumplir que, v=ru, donde -

\infty\leqr\leq\infty. Aplicando la condición vectorial se tiene que:

x-x_0=rh,

y-y_0=rk,

y al realizar un poco de álgebra se obtiene:

x=x_0+rh,

y=y_0+rk.

Estas dos ecuaciones son las que describen perimétricamente a cualquier punto de la recta L.

7. Expresión de la recta en forma general

La recta se expresa comúnmente en forma general como, Ax+By+C=0. La forma general puede

relacionarse con la forma paramétrica al observar que, r=\frac{x-x_0}{h}=\frac{y-y_0}{k},

de donde se puede realizar un poco de álgebra hasta obtener que:

kx-hy+(hy_0-kx_0)=0

Al comparar la forma general con la ecuación anterior se observa que,

A=k;\ B=-h;\ C=hy_0-kx_0.

8. Recta que pasa por dos puntos

Page 18: Diseño Instruccional Matemáticas. “Trigonometría y ...conacyt.upemor.edu.mx/informe3/archivos/PropiedadInt...Diseño Instruccional Matemáticas.“Trigonometría y Geometría

18

Cuando una recta pasa por dos puntos, se genera la geometría que se muestra en la figura

siguiente. Al considerar que la recta pasa por los puntos P_1(x_1,y_1) y P_2(x_2,y_2),

entonces si se considera un punto P(x,y) de la recta, el vector u y v se expresan como:

u=(x_2-x_1,y_2-y_1 );\ v=(x-x_1,y-y_1);

Por lo tanto, la pendiente de la recta considerando ambos vectores es:

m_L=\frac{y-y_1}{x-x_1}=\frac{y_2-y_1}{x_2-x_1},

y al realizar un poco de álgebra se obtiene:

y-y_1=\frac{y_2-y_1}{x_2-x_1}(x-x_1)

9. Ecuación de la recta al considerar la pendiente y un punto

En caso de que sea conocida la pendiente de la recta y un punto, la ecuación de la recta se

puede obtener al considerar la figura siguiente, si observamos que, cualquier punto P(x,y)

genera un vector w con el punto P_1(x_1,y_1) como:

\stackrel{\rightarrow}{P_1P}=\stackrel{\rightarrow}{u}=(x-x_1,y-y_1),

Por lo tanto, la pendiente se encuentra como:

m_L=\frac{y-y_1}{x-x_1},

a partir esta ecuación se obtiene que:

y-y_1=m_L(x-x_1).

Esta ecuación es la ecuación buscada que describe la recta.

Page 19: Diseño Instruccional Matemáticas. “Trigonometría y ...conacyt.upemor.edu.mx/informe3/archivos/PropiedadInt...Diseño Instruccional Matemáticas.“Trigonometría y Geometría

19

10. Ecuación de la recta al considerar la pendiente y la ordena al origen

Un caso particular de la ecuación de pendiente-punto es la aplicación cuando el punto es el

cruce de la recta con el eje y, considerando el punto como P_1(0,b), y a la pendiente se le

considera m_L=m, con lo cual, la ecuación de la recta es:

y-b=m(x-0),

lo cual es común expresar como:

y=m_ x+b.

La gráfica de este caso se puede ver en la figura siguiente.

Una correspondencia entre expresiones entre la expresión general de la recta y la expresión

de ordenada al origen se puede realizar al iniciar con la expresión general Ax+By+C=0, después

de realizar un poco de álgebra se obtiene que:

y=-\frac{A}{B}x+(-\frac{C}{B}),

Page 20: Diseño Instruccional Matemáticas. “Trigonometría y ...conacyt.upemor.edu.mx/informe3/archivos/PropiedadInt...Diseño Instruccional Matemáticas.“Trigonometría y Geometría

20

Por lo tanto, por comparación entre coeficientes se obtiene que:

m=-\frac{A}{B};\ b=-\frac{C}{B}.

11. Ecuación de la recta en forma simétrica

La ecuación de la recta que en forma simétrica considera que la recta intercepta a los ejes x y

y, como se muestra en la siguiente figura.

Al considerar la ecuación para la recta a la que pertenecen dos puntos, P_1(a,0) y P_2(0,b), se

obtiene el siguiente desarrollo:

y-0=\frac{b-0}{0-a}(x-a),

-ay=bx-ab,

bx+ay=ab,

\frac{x}{a}+\frac{y}{b}=1.

La relación,

\frac{x}{a}+\frac{y}{b}=1,

es la expresión analítica de la recta en forma simétrica.

12. Distancia punto recta

Toda recta L se puede visualizar como una sucesión de puntos P(x,y), los cuales satisfacen la

ecuación,

Ax+By+C=0,

o bien, la ecuación,

y=mx+b,

donde se ha considerado,

Page 21: Diseño Instruccional Matemáticas. “Trigonometría y ...conacyt.upemor.edu.mx/informe3/archivos/PropiedadInt...Diseño Instruccional Matemáticas.“Trigonometría y Geometría

21

m=-\frac{A}{B};\:\:b=-\frac{C}{B}.

Un punto exterior a una recta, tiene una distancia a cualquier punto P(x,y) de una recta.

Aunque, la distancia d se define como, “La distancia mínima de un punto P_e(x_e,y_e) exterior

a un punto P(x,y), es decir, la distancia d mínima del segmento \overline{PP_e}.

Para deducir la ecuación con la que se evalúa d, se ha construido la geometría de la figura.

Para deducir la ecuación que evalúa la distancia d se supondrá que los datos conocidos son, la

ecuación de la recta en la forma, y=mx+b, y el punto externo P_e(x_e,y_e). Para obtener la

distancia d se requiere un punto conocido, y un vector normal unitario. Como punto conocido

se usa al punto de la recta que tiene como valor en las abscisas a x=x_e, por lo tanto, el punto

conocido es, P_v(x_e,y(x_e)), por lo tanto, el vector V es,

V=(x_e-x_e,y_e-y(x_e ))=(0,y_e-mx_e-b),

donde se ha usado que la ecuación de la recta es y=mx+b, de donde, y(x_e) es,

y(x_e)=mx_e+b. Ahora un vector que es paralelo a la recta es el vector T=(1,m), por lo tanto,

un vector normal a este vector es N=(-m,1). Esto se infiere de la condición de

perpendicularidad que cumple T\circN=0, lo cual se puede verificar para nuestros vectores.

Utilizando estos vectores la distancia d es la componente del vector V sobre el vector normal

unitario, que es.

\widehat{N}=\frac{N}{\left\|N\right\|}.

Con esto, la distancia d se obtiene por,

d=\left|V\cdot\widehat{N}\right|=\left|V\cdot\left(\frac{N}{\left\|N\right\|}\right)\right|=\le

ft|\frac{V\cdot N}{\left\|N\right\|}\right|=\frac{\left|V\cdot

N\right|}{\left|\left\|N\right\|\right|}.

d=\frac{\left|\left(0,y_e-mx_e-b\right)\left(-

m,1\right)\right|}{\left|\sqrt{1+m^2}\right|}=\frac{\left|y_e-mx_e-

b\right|}{\left|\sqrt{1+m^2}\right|},

Page 22: Diseño Instruccional Matemáticas. “Trigonometría y ...conacyt.upemor.edu.mx/informe3/archivos/PropiedadInt...Diseño Instruccional Matemáticas.“Trigonometría y Geometría

22

d=\frac{\left|y_e-mx_e-b\right|}{\sqrt{1+m^2}}.

Solo para remarcar, la distancia se evaluará por,

d=\displaystyle{\frac{\left|y_e-mx_e-b\right|}{\sqrt{1+m^2}}}.

Si analizamos la relación vemos que, se utilizan los valores del punto externo x_e y y_e, y de la

ecuación de la recta se utilizan los parámetros, m y b.

En la literatura es más común utilizar la ecuación de la recta en forma general, por lo tanto, se

obtendrá una expresión para obtener la distancia considerando esa ecuación.

Recordemos que si se tiene la ecuación general de la recta en la forma, Ax+By+C=0, la

pendiente y ordenada al origen se relacionan por:

m=-\frac{A}{B};\:\:b=-\frac{C}{B},

valores que se sustituyen en la relación de la distancia d, y se obtiene,

d=\frac{\left|y_e-mx_e-b\right|}{\sqrt{1+m^2}}=\frac{\left|y_e-\left(-\frac{A}{B}\right)x_e-

\left(-\frac{C}{B}\right)\right|}{\sqrt{1+\left(-\frac{A}{B}\right)^2}}=\frac{\left|\frac{B}{B}y_e-

\left(-\frac{A}{B}\right)x_e-\left(-\frac{C}{B}\right)\right|}{\sqrt{\frac{A^2+B^2}{B^2}},

d=\frac{\frac{1}{\left|B\right|}\left|By_e+Ax_e+C\right|}{\frac{1}{\left|B\right|}\sqrt{A^2+B^

2}},

d=\frac{\left|By_e+Ax_e+C\right|}{\sqrt{A^2+B^2}}.

Esta ecuación se puede reescribir como,

d=\displaystyle{\frac{\left|Ax_e+By_e+C\right|}{\sqrt{A^2+B^2}}}.

Ésta última relación utiliza la ecuación general de la recta donde se le sustituyen los valores de

las coordenadas externas, y se utilizan los coeficientes A y B de la misma ecuación general de

la recta.

13. Bisectriz

La recta bisectriz L_b de un ángulo, lo divide en dos partes iguales. Además se caracteriza

porque cualquier punto de esta recta tiene la misma distancia en valor absoluto con la recta

superior e inferior, como se muestra en la figura siguiente. Debemos aclarar que cuando un

punto está por debajo de la recta, la distancia resulta negativa, y es la función valor absoluto lo

que nos genera el valor positivo. Cuando el punto está arriba de la recta la distancia resulta

positiva. Por lo tanto, esto aplicado a la bisectriz, en la figura nos da la condición de la

distancia del punto P(x,y) de la recta bisectriz a cada recta del ángulo, cuando las ecuaciones

de las rectas L y L' están expresadas en la forma general, se tiene que,

Page 23: Diseño Instruccional Matemáticas. “Trigonometría y ...conacyt.upemor.edu.mx/informe3/archivos/PropiedadInt...Diseño Instruccional Matemáticas.“Trigonometría y Geometría

23

d_2=-d_1,

Es decir,

\displaystyle{\frac{Ax+By+C}{\sqrt{A^2+B^2}}=-\frac{A'x+B'y+C'}{\sqrt{(A')^2+(B')^2}}},

si se definen como,

\displaystyle{\vartheta=\sqrt{A^2+B^2}},

\displaystyle{\vartheta'=\sqrt{(A')^2+(B')^2}},

la ecuación de la recta se expresa como,

(\vartheta'A+\vartheta A')x+(\vartheta'B+\vartheta B')y+(\vartheta'C+\vartheta C')=0.

que es la ecuación que describe a la recta bisectriz.

14. Intercepción entre dos rectas

Dos rectas no paralelas se interceptan en un punto P_0(x_0,y_0). Éste se encuentra al resolver

de forma simultánea las ecuaciones de las rectas L_1 y L_2, que son:

A_1x+B_1y=C_1,

A_2x+B_2y=C_2.

Dado que el punto pertenece a ambas rectas, debe ser solución del sistema, es decir, se debe

cumplir que,

A_1x_0+B_1y_0=C_1,

A_2x_0+B_2y_0=C_2.

Existen varios procedimientos para encontrar x_0 y y_0, no obstante, nosotros para obtener

estos usaremos las siguientes ecuaciones:

\displaystyle{x_0=\frac{C_1B_2-C_2B_1}{A_1B_2-A_2B_1}},

\displaystyle{y_0=\frac{C_2A_1-C_1A_2}{A_1B_2-A_2B_1}}.

Page 24: Diseño Instruccional Matemáticas. “Trigonometría y ...conacyt.upemor.edu.mx/informe3/archivos/PropiedadInt...Diseño Instruccional Matemáticas.“Trigonometría y Geometría

24

De esta forma se obtiene el punto de intercepción P_0(x_0,y_0).

15. Propiedades de los triángulos

Un triángulo es una figura geométrica que se forma por la intercepción de tres segmentos

continuos de rectas, como se muestra en la figura siguiente. Debido a su forma el triángulo

tiene también tres ángulos.

El triángulo es una figura geométrica que ha sido estudiada ampliamente, y es común que en

geometría analítica se le describa considerando que son conocidos los puntos de sus vértices.

Luego, a partir de esta información se obtienen los parámetros y ecuaciones de las rectas

características de esta figura. Los elementos que se describen son, las ecuaciones de las rectas

que contienen a los segmentos que forman el triángulo, la ubicación de los vértices, los valores

de los ángulos, las ecuaciones de las bisectrices, las ecuaciones de las medianas, las

ecuaciones de las mediatrices, el punto de intercepción de las bisectrices, el punto de

intercepción de las mediatrices, el punto de intercepción de las medianas, el punto de

intercepción de las alturas, el área y el perímetro del triángulo.

16. Longitudes de los lados del triángulo

Las longitudes d_A, d_B y d_C de los lados de un triángulo, al aplicar la relación para obtener

la distancia entre dos puntos, con los puntos que se muestran en la figura, se obtienen como:

d_A=\sqrt{(x_3-x_2)^2+(y_3-y_2)^2}

d_B=\sqrt{(x_3-x_1)^2+(y_3-y_1)^2}

d_C=\sqrt{(x_2-x_1)^2+(y_2-y_1)^2}

17. Pendientes de las rectas que contienen a los segmentos del triángulo

Page 25: Diseño Instruccional Matemáticas. “Trigonometría y ...conacyt.upemor.edu.mx/informe3/archivos/PropiedadInt...Diseño Instruccional Matemáticas.“Trigonometría y Geometría

25

Las pendientes de las rectas que contienen los segmentos \overline{AB}, \overline{BC} y

\overline{AC} se evalúan usando la ecuación para la pendiente que considera el vector entre

los dos puntos de cada segmento, con esto las pendientes se evalúan como,

\displaystyle{m_{AB}=\frac{y_2-y_1}{x_2-x_1}},

\displaystyle{m_{BC}=\frac{y_3-y_2}{x_3-x_2}},

\displaystyle{m_{AC}=\frac{y_1-y_3}{x_1-x_3}}.

18. Propiedades de los triángulos

El ángulo de los vértices se evalúa considerando la relación para calcular el ángulo entre dos

rectas. Una consideración que se debe hacer es, tomar como recta 2 de la ecuación a la que

tiene mayor ángulo, al medir éste en sentido contrario a las manecillas del reloj, por supuesto,

la recta 1 será la que tiene menor ángulo con la misma consideración.

\displaystyle{\theta_A=Tan^{-1}\left(\frac{m_{AC}-m_{AB}}{1+m_{AB}m_{AC}}\right)},

\displaystyle{\theta_B=Tan^{-1}\left(\frac{m_{AB}-m_{BC}}{1+m_{AB}m_{BC}}\right)},

\displaystyle{\theta_C=Tan^{-1}\left(\frac{m_{BC}-m_{AC}}{1+m_{BC}m_{AC}}\right)}.

19. Ecuaciones de las rectas que contienen los segmentos del triángulo

Para encontrar las ecuaciones de las rectas que contienen a los segmentos que forman un

triángulo se usará la ecuación de la recta que considera dos puntos, por lo tanto, para el

segmento \overline{AB} pertenece a la recta que se describe por,

\displaystyle{y-y_1=\frac{y_2-y_1}{x_2-x_1}\left(x-x_1\right)}.

El segmento \overline{AC} pertenece a la recta que se describe por,

\displaystyle{y-y_1=\frac{y_3-y_1}{x_3-x_1}\left(x-x_1\right)}.

El segmento \overline{BC} pertenece a la recta que se describe por,

\displaystyle{y-y_2=\frac{y_3-y_2}{x_3-x_2}\left(x-x_2\right)}.

20. Ecuaciones de las bisectrices de un triángulo

La bisectriz de un ángulo es la recta que lo divide en dos partes iguales. Por lo tanto, un

triángulo tiene tres bisectrices, una para cada ángulo. Las tres bisectrices de los ángulos

internos de un triángulo se cortan en un único punto, que equidista de los lados. A este punto

se le denomina “incentro” del triángulo y este punto tiene como característica geométrica, de

ser el centro de una circunferencia inscrita dentro del triángulo. Por lo tanto, esta

circunferencia es tangente a cada uno de los lados del triángulo.

Page 26: Diseño Instruccional Matemáticas. “Trigonometría y ...conacyt.upemor.edu.mx/informe3/archivos/PropiedadInt...Diseño Instruccional Matemáticas.“Trigonometría y Geometría

26

Las coordenadas del incentro son una combinación de las coordenadas de sus vértices y las

longitudes de sus lados. Si los vértices tienen coordenadas A(x_1,y_1), B(x_2,y_2) y C(x_3,y_3)

y sus lados opuestos tienen las longitudes d_A, d_B y d_C, entonces el incentro se obtiene por

medio de la ecuación:

\displaystyle{I\left(\frac{d_Ax_1+d_Bx_2+d_Cx_3}{d_A+d_B+d_C},\frac{d_Ay_1+d_By_2+d_Cy

_3}{d_A+d_B+d_C}\right)=I\left(x_1,y_1\right)}

Es decir,

\displaystyle{x_1=\frac{d_Ax_1+d_Bx_2+d_Cx_3}{d_A+d_B+d_C}},

\displaystyle{y_1=\frac{d_Ay_1+d_By_2+d_Cy_3}{d_A+d_B+d_C}}.

Una vez que conocemos el incentro nos podemos ayudar con este punto para obtener las

ecuaciones de las bisectrices. Dado que el incentro pertenece a las tres bisectrices, se puede

usar este punto para obtener la ecuación de la bisectriz, al considerar que la bisectriz contiene

al vértice y el incentro. A partir de esto, la bisectriz del ángulo del punto A se obtiene como,

\displaystyle{y-y_I=\frac{y_1-y_I}{x_1-x_I}\left(x_x_I\right)},

la bisectriz del ángulo del punto B se obtiene como,

\displaystyle{y-y_I=\frac{y_2-y_I}{x_2-x_I}\left(x_x_I\right)},

la bisectriz del ángulo del punto C se obtiene como,

\displaystyle{y-y_I=\frac{y_3-y_I}{x_3-x_I}\left(x_x_I\right)},

Los segmentos de las bisectrices, el incentro, y la circunferencia inscrita en el triángulo se

muestran en la siguiente figura.

Page 27: Diseño Instruccional Matemáticas. “Trigonometría y ...conacyt.upemor.edu.mx/informe3/archivos/PropiedadInt...Diseño Instruccional Matemáticas.“Trigonometría y Geometría

27

21. Medianas del triángulo

Las medianas como se muestra en la siguiente figura, son las tres rectas que unen cada vértice

del triángulo con el centro del lado opuesto. Las tres medianas se cortan en un único punto (G

en la figura siguiente) llamado centro de gravedad o centro de masa del triángulo.

Los puntos medios se encuentran por las ecuaciones que considera que un segmento se divide

en dos partes iguales, por lo tanto, para el punto D se tiene,

\displaystyle{x_{AB}=\frac{x_1+x_2}{2}},

\displaystyle{y_{AB}=\frac{y_1+y_2}{2}}.

para el punto E se tiene,

\displaystyle{x_{BC}=\frac{x_2+x_3}{2}},

\displaystyle{y_{BC}=\frac{y_2+y_3}{2}}.

para el punto F se tiene,

\displaystyle{x_{AB}=\frac{x_1+x_2}{2}},

\displaystyle{y_{AB}=\frac{y_1+y_2}{2}}.

Al utilizar estos puntos se encuentran las ecuaciones de las rectas, con el uso de la ecuación de

la recta que considera dos puntos conocidos, así el segmento \overline{AE} está contenida en

la recta,

\displaystyle{y-y_1=\frac{y_{BC}-y_1}{x_{BC}-x_1}\left(x-x_1\right)}.

el segmento \overline{BF} está contenida en la recta,

\displaystyle{y-y_2=\frac{y_{AC}-y_2}{x_{AC}-x_2}\left(x-x_2\right)}

el segmento \overline{CD} está contenida en la recta,

Page 28: Diseño Instruccional Matemáticas. “Trigonometría y ...conacyt.upemor.edu.mx/informe3/archivos/PropiedadInt...Diseño Instruccional Matemáticas.“Trigonometría y Geometría

28

\displaystyle{y-y_3=\frac{y_{AB}-y_3}{x_{AB}-x_3}\left(x-x_3\right)}

El punto G, se obtiene por la solución simultánea de dos de las ecuaciones para los segmentos

\overline{AE}, \overline{BF} y \overline{CD}.

22. Mediatrices

La mediatriz de un segmento es la recta perpendicular a dicho segmento trazada a partir de su

punto medio. Por la propiedad antes mencionada, en todo triángulo ABC como se muestra en

la siguiente figura, las mediatrices de sus tres lados concurren en un mismo punto, llamado el

circuncentro del triángulo (punto O en la figura). Dicho punto equidista de los vértices del

triángulo. La circunferencia del centro O y de radio OA, pasa por los otros dos vértices del

triángulo. Se dice que dicha circunferencia es circunscrita al triángulo y que el triángulo está

inscrito en la circunferencia.

Las ecuaciones de las mediatrices se deducen al saber que son perpendiculares al segmento

en el punto medio, esta condición nos hace inferir que la recta M_{AB} tiene pendiente que

cumple, (m_{AB})m_{AB\bot}=-1, dado que,

\displaystyle{m_{AB}=\frac{y_2-y_1}{x_2-x_1}},

entonces,

\displaystyle{m_{AB\bot}=-\frac{x_2-x_1}{y_2-y_1}}

El punto medio D es,

Page 29: Diseño Instruccional Matemáticas. “Trigonometría y ...conacyt.upemor.edu.mx/informe3/archivos/PropiedadInt...Diseño Instruccional Matemáticas.“Trigonometría y Geometría

29

\displaystyle{x_{AB}=\frac{x_1+x_2}{2}},

\displaystyle{y_{AB}=\frac{y_1+y_2}{2}}.

Utilizando esto, la ecuación de la mediatriz M_{AB} es,

\displaystyle{y-\left(\frac{y_1+y_2}{2}\right)=-\frac{x_2+x_1}{y_2-y_1}\left(x-

\left(\frac{x_1+x_2}{2}\right)\right)}.

La recta M_{BC} tiene pendiente que cumple, (m_{BC})m_{bc\bot}=-1 dado que,

\displaystyle{m_{BC}=\frac{y_3-y_2}{x_3-x_2}},

entonces,

\displaystyle{m_{BC\bot}=-\frac{x_3-x_2}{y_3-y_2}}.

El punto medio E es,

\displaystyle{x_{BC}=\frac{x_2+x_3}{2}},

\displaystyle{y_{BC}=\frac{y_2+y_3}{2}}.

Utilizando esto, la ecuación de la mediatriz M_{BC} es,

\displaystyle{y-\left(\frac{y_2+y_3}{2}\right)=-\frac{x_3-x_2}{y_3-y_2}\left(x-

\left(\frac{x_2+x_3}{2}\right)\right)}.

La recta M_{AC} tiene pendiente que cumple, (m_{AC})m_{AC\bot}=-1 dado que,

\displaystyle{m_{AC}=\frac{y_1-y_3}{x_1-x_3}},

entonces,

\displaystyle{m_{AC\bot}=-\frac{x_1-x_3}{y_1-y_3}}.

El punto medio F es,

\displaystyle{x_{AC}=\frac{x_1+x_3}{2}},

\displaystyle{y_{AC}=\frac{y_1+y_3}{2}}.

Utilizando esto, la ecuación de la mediatriz M_{AC} es,

\displaystyle{y-\left(\frac{y_1+y_3}{2}\right)=-\frac{x_1-x_3}{y_1-y_3}\left(x-

\left(\frac{x_1+x_3}{2}\right)\right)}.

El punto 0(h,k) se encuentra por la solución simultánea de dos de las tres ecuaciones de las

mediatrices, M_{AB}, M_{BC} y M_{AC}.

Page 30: Diseño Instruccional Matemáticas. “Trigonometría y ...conacyt.upemor.edu.mx/informe3/archivos/PropiedadInt...Diseño Instruccional Matemáticas.“Trigonometría y Geometría

30

23. Alturas de un triángulo

La altura de un triángulo con respecto de un lado del triángulo, es la distancia más corta entre

la recta que contiene al lado y el vértice opuesto. Por lo tanto, un triángulo tiene tres alturas

que en la figura h_A, h_B y h_C. Las alturas se interceptan en un punto llamado ortocentro

que en la figura se muestra por H. Debemos comentar que en todo triángulo, al menos una de

las alturas se encuentra dentro del triángulo, y que la altura de mayor longitud es la

correspondiente al lado menor del triángulo.

Para obtener la ecuación de la recta que contiene a “la altura” se tiene que considerar el

punto vértice de donde se genera la altura y la pendiente de la recta que contiene a la altura,

al saber que es perpendicular a la recta que contiene al segmento del lado del triángulo

opuesto al vértice. Para el caso de nuestra figura, la recta H_A, el punto vértice es A(x_1,y_1),

y la pendiente es,

\displaystyle{m_{BC\bot}=-\frac{x_3-x_2}{y_3-y_2}},

con esto, la ecuación de la recta que contiene a esta altura es,

\displaystyle{y-y_1=-\frac{x_3-x_2}{y_3-y_2}\left(x-x_1\right)}.

Para encontrar la longitud de la altura se considera la ecuación de la recta del segmento

contrario al vértice y en esta ecuación se aplica las coordenadas del vértice como se mostró en

la fórmula de la distancia de un punto a una recta. En este caso, la ecuación de la recta del

segmento \overline{BC} es,

\displaystyle{y-y_2=-\frac{y_3-y_2}{x_3-x_2}\left(x-x_2\right)},

Page 31: Diseño Instruccional Matemáticas. “Trigonometría y ...conacyt.upemor.edu.mx/informe3/archivos/PropiedadInt...Diseño Instruccional Matemáticas.“Trigonometría y Geometría

31

o bien,

\left(y_3-y_2\right)x-\left(x_3-x_2\right)y+\left(x_3-x_2\right)y_2-\left(y_3-y_2\right)x_2=0,

al considerar esta ecuación la altura se obtiene como,

\displaystyle{h_A=\frac{\left|\left(y_3-y_2\right)x_1-\left(x_3-x_2\right)y_1+\left(x_3-

x_2\right)y_2-\left(y_3-y_2\right)x_2\right|}{\sqrt{\left(y_3-y_2\right)^2+\left(x_3-

x_2\right)^2}}},

La recta H_C, el punto vértice es C(x_3,y_3), y la pendiente es,

\displaystyle{m_{AB\bot}=-\frac{x_2-x_1}{y_2-y_1}},

con esto, la ecuación de la recta que contiene a esta altura H_C es,

\displaystyle{y-y_1=-\frac{x_2-x_1}{y_2-y_1}\left(x-x_3\right)}.

Para encontrar la altura h_C se considera la ecuación de la recta del segmento contrario al

vértice C, y en esta ecuación se aplica las coordenadas de este vértice, como fue mostrado en

la fórmula de la distancia de un punto a una recta. En este caso, la ecuación de la recta del

segmento \overline{AB} es,

\displaystyle{y-y_1=-\frac{y_2-y_1}{x_2-x_1}\left(x-x_1\right)}

o bien,

\left(y_2-y_1\right)x-\left(x_2-x_1\right)y+\left(x_2-x_1\right)y_1-\left(y_2-y_1\right)x_1=0,

al considerar esta ecuación la altura h_C se obtiene como,

\displaystyle{h_C=\frac{\left|\left(y_2-y_1\right)x_2-\left(x_2-x_1\right)y_3+\left(x_2-

x_1\right)y_1-\left(y_2-y_1\right)x_1\right|}{\sqrt{\left(y_2-y_1\right)^2+\left(x_2-

x_1\right)^2}}}.

La recta H_B, el punto vértice es B(x_2,y_2), y la pendiente es,

\displaystyle{m_{AC\bot}=-\frac{x_1-x_3}{y_1-y_3}},

con esto, la ecuación de la recta que contiene a esta altura H_B es,

\displaystyle{y-y_2=-\frac{x_1-x_3}{y_1-y_3}\left(x-x_2\right)}.

Para encontrar la altura h_B se considera la ecuación de la recta del segmento contrario al

vértice B(x_2,y_2), y en esta ecuación se aplica las coordenadas de este vértice, como fue

mostrado en la fórmula de la distancia de un punto a una recta. En este caso, la ecuación de la

recta del segmento \overline{AC} es,

Page 32: Diseño Instruccional Matemáticas. “Trigonometría y ...conacyt.upemor.edu.mx/informe3/archivos/PropiedadInt...Diseño Instruccional Matemáticas.“Trigonometría y Geometría

32

\displaystyle{y-y_1=-\frac{y_3-y_1}{x_3-x_1}\left(x-x_1\right)}

o bien,

\left(y_3-y_1\right)x-\left(x_3-x_1\right)y+\left(x_3-x_1\right)y_1-\left(y_3-y_1\right)x_1=0,

al considerar esta ecuación la altura se obtiene como,

\displaystyle{h_B=\frac{\left|\left(y_3-y_1\right)x_2-\left(x_3-x_1\right)y_2+\left(x_3-

x_1\right)y_1-\left(y_3-y_1\right)x_1\right|}{\sqrt{\left(y_3-y_1\right)^2+\left(x_3-

x_1\right)^2}}}.

La intercepción de las alturas se puede obtener por la solución simultánea de dos de las tres

ecuaciones de las rectas H_A, H_B y H_C.

Debemos comentar que en un triángulo equilátero, las medianas coinciden con las mediatrices

de los lados, con las alturas del triángulo, y con las bisectrices de los tres ángulos.

24. El perímetro y el área del triángulo

El perímetro de un triángulo es la suma de las longitudes de sus lados. Por lo tanto, es,

P=d_A+d_B+d_C.

El área de un triángulo se encuentra por la conocida fórmula,

\displaystyle{A=\frac{b \cdot h}{2}},

en este caso, puede ser,

\displaystyle{A=\frac{d_Ah_A}{2}},

\displaystyle{A=\frac{\left(\sqrt{\left(x_3-x_2\right)^2+\left(y_3-

y_2\right)^2}\right)\left(\frac{\left|\left(y_3-y_2\right)x_1-\left(x_3-x_2\right)y_1+\left(x_3-

x_2\right)y_2-\left(y_3-y_2\right)x_2\right|}{\sqrt{\left(x_3-x_2\right)^2+\left(y_3-

y_2\right)^2}}\right)}{2}},

\displaystyle{A=\frac{\left|\left(y_3-y_2\right)x_1-\left(x_3-x_2\right)y_1+\left(x_3-

x_2\right)y_2-\left(y_3-y_2\right)x_2\right|}{2}}.

O bien, al considerar d_B y h_C, o d_C y h_B, se obtiene,

\displaystyle{A=\frac{\left|\left(y_3-y_1\right)x_2-\left(x_3-x_1\right)y_2+\left(x_3-

x_1\right)y_1-\left(y_3-y_1\right)x_1\right|}{2}},

\displaystyle{A=\frac{\left|\left(y_2-y_1\right)x_3-\left(x_2-x_1\right)y_3+\left(x_2-

x_1\right)y_1-\left(y_2-y_1\right)x_1\right|}{2}}.

Page 33: Diseño Instruccional Matemáticas. “Trigonometría y ...conacyt.upemor.edu.mx/informe3/archivos/PropiedadInt...Diseño Instruccional Matemáticas.“Trigonometría y Geometría

33

Con esto se ha definido las propiedades de un triángulo

25. Problemas

Módulo III. Lugares geométricos I: Circunferencia y elipse

En esta lección repasaremos uno de los conceptos más importantes en la geometría analítica, el de

“lugar geométrico”. También con especial atención estudiaremos dos lugares geométricos

notables: la circunferencia y la elipse. Así, los objetivos de la lección se pueden resumir en los

siguientes tres puntos:

1. Hacer conciente el significado de ``lugar geométrico'' y el proceso para representarlo

matemáticamente.

2. Revisar la representación matemática de las circunferencias utilizando la herramienta vectorial.

3. Revisar la representación matemática de las elipses utilizando la herramienta vectorial.

1. Sobre los lugares geométricos

Una definición simple de lugar geométrico nos dice que es un conjunto de puntos que

obedecen alguna restricción geométrica. Los lugares geométricos, generalmente permiten

derivar relaciones matemáticas entre los puntos que los conforman y usualmente estas

ecuaciones son la forma más práctica de caracterizarlos. Entonces el problema matemático

asociado a este concepto es comúnmente encontrar las ecuaciones matemáticas que los

definen.

Supongamos ahora, como ejemplo, que se quiere encontrar la ecuación que representa al

lugar geométrico de los puntos que equidistan de dos puntos fijos A=(1,4) y B=(3,1). Para

hacerlo se puede usar una construcción como la que se muestra en la figura siguiente:

Page 34: Diseño Instruccional Matemáticas. “Trigonometría y ...conacyt.upemor.edu.mx/informe3/archivos/PropiedadInt...Diseño Instruccional Matemáticas.“Trigonometría y Geometría

34

En la figura se trazaron los vectores u y v, asociándolos a los puntos A y B respectivamente. Un

vector w=(x,y) que apunte a cualquier punto contenido en el lugar geométrico debe cumplir

(como también puede verse en la figura) que sus distancias a los puntos A y B son iguales. Esta

condición se puede expresar usando operaciones vectoriales sobre los vectores z y s, los

cuales, como se observa, apuntan de D a A y de D a B respectivamente. La condición de

equidistancia se puede escribir entonces así:

\begin{displaymath} \vert z\vert=\vert s\vert, \end{displaymath}

(1)

o bien

\begin{displaymath} \sqrt{z \cdot z}=\sqrt{s \cdot s} \end{displaymath}

(2)

que equivale a igualar los argumentos de las raíces:

\begin{displaymath} z \cdot z= s \cdot s \end{displaymath}

(3)

Pero como podemos ver se cumple que z=u-w y s=v-w por lo que sustituyendo en 3,

\begin{displaymath} (u-w) \cdot (u-w) - (v-w) \cdot (v-w) =0. \end{displaymath}

(4)

Esta última ecuación es la relación vectorial que obedecen los puntos del lugar geométrico.

Para obtener una relación cartesiana entre sus coordenadas, sustituimos los valores de

u=(1,4), v=(3,1) y w=(x,y) quedando:

\begin{displaymath} (1-x)^2+(4-y)^2-(3-x)^2-(1-y)^2=0.\end{displaymath}

(5)

Desarrollando los binomios al cuadrado y simplificando nos lleva directamente a la ecuación:

\begin{displaymath} 4x-6y+7=0, \end{displaymath}

(6)

que es la ecuación de la recta que se muestra en color rojo. Noten que en la construcción es

posible mover el punto D arrastrándolo con el ratón y al mismo tiempo verificar que cualquier

punto en la recta cumple la condición pedida. También es interesante jugar con la

construcción variando A y B notando que se puede obtener la solución para cualquier par de

puntos. Esto quiere decir que todas las rectas están descritas por un lugar geométrico como

Page 35: Diseño Instruccional Matemáticas. “Trigonometría y ...conacyt.upemor.edu.mx/informe3/archivos/PropiedadInt...Diseño Instruccional Matemáticas.“Trigonometría y Geometría

35

este. En otras palabras una recta también es ``el lugar geométrico de los puntos que

equidistan de dos puntos fijos''.

El ejemplo tiene dos enseñanzas: primero la manera como enunciar un lugar geométrico y

segundo la manera como encontrar la relación matemática que lo describe. Así el proceso

para encontrar la ecuación de un lugar geométrico puede resumirse como sigue:

1. Entender la definición propuesta.

2. Traducir a una construcción geométrica (basada en vectores o en cualquier otra técnica).

3. Obtener la relación matemática más simple que describe al lugar geométrico. Esto es

mediante una simplificación algebraica.

Elige ahora el siguiente paso, que puede ser ir a una serie de ejercicios de práctica o de

continuar con la sección de circunferencia.

2. La circunferencia

Esta sección la empezaremos con un problema y discutiremos su solución:

El problema consiste en encontrar la ecuación del lugar geométrico de los puntos que están a

una distancia igual a 3 del punto A=(1,1).

Construcción: Usando vectores se puede hacer la siguiente construcción:

El vector u está asociado al punto A, el vector v es apunta al punto C, el cual pertenece al lugar

geométrico. El vector w va de A a C y por tanto cumple que w=v-u. La línea roja es el lugar

Page 36: Diseño Instruccional Matemáticas. “Trigonometría y ...conacyt.upemor.edu.mx/informe3/archivos/PropiedadInt...Diseño Instruccional Matemáticas.“Trigonometría y Geometría

36

geométrico buscado, que claramente es una circunferencia. Prueba a mover con el ratón el

punto C, cualquier punto verifica la condición de estar a 3 unidades del punto A.

Solución matemática: Cualquier punto en el lugar geométrico esta a una distancia 3 de A, esto

en lenguaje de vectores se escribe así de simple:

|w|=3.

(7)

Pero por construcción w=v-u, entonces:

|v-u|=3,

(8)

que usando la relación entre el tamaño y el producto punto se puede escribir:

\sqrt{(v-u)\cdot(v-u)}=3.

(9)

Elevando al cuadrado la ecuación anterior también se puede escribir así:

\begin{displaymath} (v-u)\cdot(v-u)=9. \end{displaymath}

(10)

Esta última ecuación es la relación vectorial entre los puntos del lugar geométrico. La relación

cartesiana se puede hacer sustituyendo u=(1,1) y v=(x,y) de modo que:

\begin{displaymath} (x-1)^2+(y-1)^2=9. \end{displaymath}

(11)

Que es una relación más familiar.

Prueben a mover el punto A en le construcción. Es claro que la circunferencia sigue siendo la

misma pero el lugar geométrico es diferente. Sin embargo la ecuación (10) es la misma solo

hay que cambiar el valor al vector u. Si este fuera un valor cualquiera u=(x_0,y_0) la ecuación

cartesiana sería

(x-x_0)^2+(y-y_0)^2=9.

(12)

Finalmente la distancia de los puntos del lugar geométrico al punto A coincide con el radio de

la circunferencia, si lo definimos con R entonces las ecuaciones vectoriales y cartesianas de la

construcción geométrica quedarían así:

Page 37: Diseño Instruccional Matemáticas. “Trigonometría y ...conacyt.upemor.edu.mx/informe3/archivos/PropiedadInt...Diseño Instruccional Matemáticas.“Trigonometría y Geometría

37

\begin{displaymath} (v-u)\cdot(v-u)=R^2, \end{displaymath}

(13)

y

\begin{displaymath} (x-x_0)^2+(y-y_0)^2=R^2. \end{displaymath}

(14)

En general, una circunferencia es entonces el lugar geométrico de los puntos que equidistan a

una distancia R de un punto fijo llamado centro.

3. Elipses

Esta sección se puede iniciar enunciando la definición de elipse: ``una elipse es el lugar

geométrico de los puntos del plano tales que la suma de las distancias a dos puntos fijos

llamados focos es una constante''.

Gráficamente la construcción siguiente muestra los puntos que forman a una elipse, definida

por los puntos F1 y F2 (que son los focos) y la suma de distancia S (en este caso se escogió

S=9).

Observen en la construcción geométrica lo siguiente: el valor de S puede escogerse moviendo

el deslizador con el ratón, prueben cambiarlo y noten como cambia la elipse. También pueden

moverse los focos para cambiar la elipse. El punto B es parte del lugar geométrico y noten que

si se suman los tamaños de los vectores que lo unen con los focos u y v coincide con el valor

de S. Prueben mover B y noten que la suma siempre es S. Otra observación importante es que

Page 38: Diseño Instruccional Matemáticas. “Trigonometría y ...conacyt.upemor.edu.mx/informe3/archivos/PropiedadInt...Diseño Instruccional Matemáticas.“Trigonometría y Geometría

38

hay un valor de S debajo del cual las elipses comienzan a dibujarse como rectas por que el

lugar geométrico realmente no existe. Esto sucede cuando S es menor que la distancia entre

los focos. En otras palabras, la suma de las distancias de cualquier punto en la elipse a los

focos debe ser siempre mayor a la distancia focal.

También es importante destacar que en la elipse se pueden ver dos ejes de simetría uno en la

dirección más larga y el otro perpendicular a éste. Cada uno de estos ejes divide a la figura en

partes iguales. De manera natural para distinguirlos se les llama eje mayor y eje menor. Los

puntos extremos de la elipse sobre los ejes se les llama vértices y el punto de intersección de

ellos es el centro de la figura. Finalmente un semieje de la elipse se define como un segmento

formado entre el centro y alguno de los vértices.

Los vectores u y v, de la figura son la base para deducir la ecuación que describe este lugar

geométrico pues su definición puede traducirse matemáticamente así:

\begin{displaymath} \vert u\vert+\vert v\vert=S. \end{displaymath}

(15)

Pero u y v son vectores relativos entre B y los focos F1 y F2. Para medirlos desde el origen

podemos usar los vectores auxiliares w, l y z, dibujados en líneas punteadas. En función de

estos u=l-w y v=l-z. Noten que l=(x,y) es cualquier punto en el lugar geométrico y en general

las coordenadas de los focos coinciden con w=(x_1,y_1) y z=(x_2,y_2). Entonces en general

una ecuación vectorial anterior se ve así:

\begin{displaymath} \vert l-w\vert+\vert l-z\vert=S, \end{displaymath}

(16)

o bien:

\begin{displaymath} \vert(x-x_1,y-y_1)\vert+\vert(x-x_2,y-y_2)\vert=S, \end{displaymath}

(17)

que equivale a la siguiente ecuación cartesiana:

\sqrt{(x-x_1)^2+(y-y_1)^2}+\sqrt{(x-x_2)^2+(y-y_2)^2}=S.

(18)

Esta ya es la ecuación cartesiana de la elipse pero en una versión poco elegante. Para

convertirla en una ecuación mas estandarizada hay que simplificar y el proceso algebraico para

hacerlo implica un buen rato de algebra. El proceso consiste tomar la ecuación elevarla al

cuadrado, desarrollar binomios al cuadrado, simplificar, volver a elevar al cuadrado y

simplificar de nuevo. El proceso es arduo y es un buen ejercicio algebraico. Sin embargo es

posible seguir un camino un poco mas heurístico partiendo de algunas simplificaciones.

Page 39: Diseño Instruccional Matemáticas. “Trigonometría y ...conacyt.upemor.edu.mx/informe3/archivos/PropiedadInt...Diseño Instruccional Matemáticas.“Trigonometría y Geometría

39

Primero planteemos una elipse con centro en el origen como la que se muestra de rojo en la

siguiente figura:

Toda elipse tiene un eje mayor y un eje menor. En este caso el eje mayor esta en dirección al

eje x. Los valores a y b que se encuentran como deslizadores en la figura corresponden al valor

de los semiejes, en el caso inicial a el mayor y b el menor. Si se mueven los deslizadores la

elipse que se dibuja cambia e incluso a puede pasar a ser el mayor y b el menor. La

construcción importante es la circunferencia azul, la cual tiene un radio que coincide con el

semieje a. El vector r' describe un punto arbitrario sobre esta circunferencia, de hecho en la

figura puede moverse a voluntad arrastrándolo con el ratón, por esto lo describiremos como

r'=(x',y'). El vector r=(x,y) se construyó con las condiciones x'=x e y'=\frac{a}{b}y. Noten que así

definido el vector r' apunta siempre a un punto en la elipse. Ahora bien, como r' apunta a la

circunferencia sus coordenadas cumplen la ecuación de la circunferencia

\begin{displaymath} x'^2+y'^2=a^2, \end{displaymath}

(19)

su se sustituyen las condiciones mencionadas la ecuación anterior queda:

\begin{displaymath} x^2+(\frac{a}{b}y)^2=a^2, \end{displaymath}

(20)

que también al dividirse toda entre a^2 puede escribiese como:

\begin{displaymath} \frac{x^2}{a^2}+\frac{y^2}{b^2}=1. \end{displaymath}

(21)

Page 40: Diseño Instruccional Matemáticas. “Trigonometría y ...conacyt.upemor.edu.mx/informe3/archivos/PropiedadInt...Diseño Instruccional Matemáticas.“Trigonometría y Geometría

40

Que corresponde a la ecuación que describe al lugar geométrico de una elipse centrada en el

origen con semiejes de medida a y <b. Si el centro de la elipse estuviera fuera del origen,

digamos en C=(x_0,y_0) el razonamiento anterior puede aplicarse idéntico pero ahora la

circunferencia tendría una ecuación:

\begin{displaymath} (x'-x_0)^2+(y'-y_0)^2=a^2, \end{displaymath}

(22)

y se debe cumplir que x'=x e y-y_0=\frac{a}{b}(y-y_0) de modo que la ecuación de la elipse

será:

\begin{displaymath} \frac{(x-x_0)^2}{a^2}+\frac{(y-y_0)^2}{b^2}=1. \end{displaymath}

(23)

Ahora puedes continuar con los ejercicios de ejemplo de esta sección.

4. Problemas

5. Cuestionario

Módulo IV. Lugares geométricos II: Parábolas e Hipérbolas

1. Parábola I

Se puede decir que cualquier conjunto de puntos, tal como la recta, la parábola, la elipse, etc.,

es un lugar geométrico \rho. El término lugar geométrico se aplica normalmente al conjunto

de todos los puntos que tienen alguna característica geométrica común.

En esta lección estudiamos a la parábola como el lugar geométrico de los puntos del plano que

equidistan de una recta y un punto fijo, donde el punto no pertenece a la recta. A esta recta se

le conoce como la Directriz d y al punto F como el Foco. La recta que contiene al foco de la

parábola y que es perpendicular a la directriz es el eje de la parábola. El punto V de la

intercesión de una parábola con su eje recibe el nombre de vértice.

2. Parábola II

En la siguiente construcción al mover el punto P se dibuja la parábola. También se muestra

que la distancia entre el punto P y el foco, comparada con la distancia de un punto sobre la

directriz al foco es la misma. Además es fácil identificar que si el punto Q no pertenece al lugar

geométrico definido por la parábola, las distancias anteriores no coinciden.

Page 41: Diseño Instruccional Matemáticas. “Trigonometría y ...conacyt.upemor.edu.mx/informe3/archivos/PropiedadInt...Diseño Instruccional Matemáticas.“Trigonometría y Geometría

41

Obtengamos la ecuación de la parábola que tiene su foco en el punto F(0,2) y cuya directriz d

es una recta con ecuación (ver figura de abajo)

y=-2,

obsérvese que U(x,y) sobre la parábola es un punto del lugar geométrico sí y solo sí

\left|u-f\right|=\left|u-w\right|,

donde W es la proyección (perpendicular) de U(x,y) sobre \rho. Entonces como U tiene las

coordenadas (x,y), F tiene como coordenadas a (0,2), y W tiene como coordenadas a (x,-2), se

tiene

\left|\left(x,y\right)-\left(0,2\right)\right| =\left|\left(x,y\right)-\left(x,-2\right)\right|,

\left|\left(x-0,y-2\right)\right|=\left|\left(x-x , y+2\right)\right|,

\sqrt{x^2+\left(y-2\right)^2} =\sqrt{0^2+\left(y+2\right)^2} ,

Page 42: Diseño Instruccional Matemáticas. “Trigonometría y ...conacyt.upemor.edu.mx/informe3/archivos/PropiedadInt...Diseño Instruccional Matemáticas.“Trigonometría y Geometría

42

x^2+\left(y-2\right)^2} =\left(y+2\right)^2 ,

x^2+y^2 - 4 y +4 = y^2 +2 y +4 ,

x^2 =8 y .

Como las raíces cuadradas involucradas en este cálculo son no negativas, los pasos de este

proceso son reversibles y la gráfica de x^2 = 8 y es la parábola requerida.

3. Parábola III

Mediante un razonamiento similar se puede demostrar que una ecuación de la parábola cuyo

foco es F(0,p) y cuya directriz es la recta d dada por y = -p es

x^2= 4 p y .

La ecuación anterior recibe el nombre de forma ordinaria de la ecuación de una parábola, con

vértice en el origen y foco sobre un eje de coordenadas. Si p&gt;0, entonces la parábola se

abre hacia arriba. Si p&lt;0, entonces la parábola se abre hacia abajo. Caso similar ocurre si

foco se encuentra en F(p,0) y cuya directriz es la recta d dada por x= -p . En este caso la

ecuación de la parábola es

y^2= 4 p x .

Si p&gt;0, entonces la parábola se abre hacia la derecha. Si p&lt;0, entonces la parábola se

abre hacia la izquierda.

Si se mueve la dirección de la directriz dejando fijo el foco entonces la parábola puede abrirse

hacia arriba o hacia abajo, hacia la derecha o izquierda tal como se observa en la siguiente

construcción.

Page 43: Diseño Instruccional Matemáticas. “Trigonometría y ...conacyt.upemor.edu.mx/informe3/archivos/PropiedadInt...Diseño Instruccional Matemáticas.“Trigonometría y Geometría

43

Mueve en la figura superior, el punto “muéveme” que está sobre la recta, la directriz y

observa los cambios:

¿Puedes identificar las distintas formas de la parábola?

Visualiza también las parábolas cuando se deja fija la directriz pero se mueve el foco F.

¿Podrías escribir la ecuación las diferentes ecuaciones de las parábolas?

4. Parábola IV

La forma general de la ecuación de la parábola con eje paralelo al eje y es

y = a x^2 + b x + c.

Esta parábola se abre hacia arriba si a&gt;0 o bien hacia abajo si a&lt;0. En la siguiente

construcción se puede observar la forma de la parábola moviendo los parámetros de la

ecuación general de la misma.

Mueve en la figura superior, los distintos parámetros a, b , y c de la ecuación de la recta y

observa los cambios:

¿Puedes identificar qué cambios provoca en la forma de la parábola cada uno de los

parámetros cuando dos de ellos se mantienen fijos?

5. Aplicaciones I

Al girar una parábola alrededor de su eje, obtenemos una superficie de revolución llamada

paraboloide. Estas superficies tienen muchas aplicaciones, principalmente en óptica y

electrónica, ya que si un rayo de luz paralelo al eje choca contra el paraboloide, entonces se

refleja hacia su foco, e inversamente, si un rayo sale del foco y choca contra el paraboloide,

entonces se refleja en la dirección de su eje.

Page 44: Diseño Instruccional Matemáticas. “Trigonometría y ...conacyt.upemor.edu.mx/informe3/archivos/PropiedadInt...Diseño Instruccional Matemáticas.“Trigonometría y Geometría

44

En la figura superior, mueve los puntos en rojo y en verde y observa los cambios:

¿Puedes identificar que independiente del punto en rojo p, los rayos que viajan paralelo al eje

del paraboloide siempre se reflejan hacia el foco F?

¿Puedes identificar que si lo rayos salen del foco F y van al paraboloide entonces se reflejan de

manera paralela a su eje?

Los rayos del sol por viajar miles de kilómetros antes de llegar a la tierra, se consideran

paralelos al incidir en un objeto por ejemplo una lente, y van al foco. Basado en esta idea,

¿podrías diseñar un concentrador solar? ¿Qué forma tendría?

Esta propiedad, conocida como la propiedad de reflexión o propiedad óptica de la parábola,

tiene muchas aplicaciones; por ejemplo, en los faros de los automóviles, las antenas

parabólicas, los telescopios, los micrófonos direccionales, etc.

6. Hipérbola I

Como hemos visto en la sección anterior, la parábola es el lugar geométrico de los puntos del

plano tales que están a la misma distancia de una recta d, conocida como directriz, y de un

punto dado F, llamado foco de la parábola. En esta lección se estudiará el lugar geométrico

\rho determinado por la diferencia de dos distancias: la hipérbola.

7. Hipérbola II

Sean F1 y F2 dos puntos en el plano, y sea k una constante tal que k &lt; dist\left(F1,F2\right).

Donde dist\left(F1,F2\right) es la distancia entre los puntos F1 y F2. Sea U un punto en el

plano, y considérese la distancia entre U y F1 y la distancia entre U y F2. Supóngase que la

diferencia entre dichas distancias es igual a k. El conjunto de los puntos del plano

Page 45: Diseño Instruccional Matemáticas. “Trigonometría y ...conacyt.upemor.edu.mx/informe3/archivos/PropiedadInt...Diseño Instruccional Matemáticas.“Trigonometría y Geometría

45

U\left(x,y\right) que cumplen esta propiedad forman una hipérbola. En la construcción de

abajo, se muestran los focos F1, F2, el eje principal, y el punto medio C que se refiere al centro

de la hipérbola. La hipérbola, a diferencia de las otras cónicas, está formada por dos partes

separadas llamadas ramas de la hipérbola.

Como puede observarse en la construcción de arriba, al mover el punto U2 se dibujan las dos

ramas de la hipérbola. Es claro que

\displaystyle{\left|dist\left(U1,F2\right)-dist\left(U1,F2\right)\right|

=\left|dist\left(U2,F1\right)-dist\left(U2,F2\right)\right| = Cte}.

Además es fácil identificar que si el punto Q no pertenece al lugar geométrico definido por la

hipérbola, la distancia \displaystyle{\left|dist\left(U2,F1\right)-dist\left(U2,F2\right)\right|} no

es una constante.

8. Hipérbola III

Considérese ahora la hipérbola (ver figura de abajo) cuyo centro es el origen, cuyos focos son

F1(c,0) y F2(-c,0), y para la cual |dist(U,F1)-dist(U,F2)| = 2a, donde a es una constante positiva

tal que a&lt;c, y U es un punto en la hipérbola. Entonces por definición, el punto U(x,y) está

sobre la hipérbola si y sólo si

|u-f1|-|u-f2|=2a,

o bien

|u-f1|-|u-f2|=\pm 2a.

Page 46: Diseño Instruccional Matemáticas. “Trigonometría y ...conacyt.upemor.edu.mx/informe3/archivos/PropiedadInt...Diseño Instruccional Matemáticas.“Trigonometría y Geometría

46

Puesto que u=(x,y), f1=(c,0), y f2(-c,0), esta última ecuación se puede escribir en la forma

\sqrt{(x-c)^2+y^2}-\sqrt{[(x-(-c)]^2+y^2} =\pm 2a.

Si ahora se \sqrt{(x+c)^2+y^2} a ambos miembros de esta ecuación, y se eleva al cuadrado a

ambos términos resultantes se obtiene

(x-c)^2+y^2=4a^2\pm 4a\sqrt{(x+c)^2+y^2}.

Si ahora se eleva al cuadrado ambos miembros de la ecuación, simplificando y factorizando el

resultado se obtiene

x^2(c^2-a^2)+a^2(a^2-c^2)=a^2y^2.

Puesto que 0&lt;a&lt;c, se sigue que a^2&lt;c^2 y c^2-a^2&gt;0. Si se hace b^2=c^2-a^2,

entonces esta ecuación se reduce a

b^2x^2-a^2y^2=a^2b^2,

que finalmente dividiendo por a^2b^2, se llega

\displaystyle{\frac{x^2}{a^2}-\frac{y^2}{b^2}=1}.

En la siguiente construcción mueve los puntos a y b, parámetros de la ecuación general de la

hipérbola, y observa los cambios:

¿Puedes observar que los cambios en ramas de las hipérbolas?

Page 47: Diseño Instruccional Matemáticas. “Trigonometría y ...conacyt.upemor.edu.mx/informe3/archivos/PropiedadInt...Diseño Instruccional Matemáticas.“Trigonometría y Geometría

47

9. Ejercicios

10. Cuestionario

Módulo V. Coordenadas polares

En esta lección prestaremos una introducción a la representación de lugares geométricos en

coordenadas polares. Esta elección de coordenadas es especialmente útil sobre todo en el estudio

de las cónicas ya que, como se verá más adelante, permite englobar a todas ellas en una sola

expresión matemática general. Así los objetivos de esta sección son los siguientes:

1. Conocer la representación de los vectores en coordenadas polares y su relación con las

coordenadas cartesianas.

2. Explorar las ecuaciones que describen a las cónicas en coordenadas polares.

3. Conocer y razonar el concepto de excentricidad.

1. Coordenadas polares

En la lección sobre los vectores se introdujo la figura siguiente y se hizo notar que un vector

además de tamaño tiene dirección y se encontró que para un vector u=(x,y) su tamaño y

ángulo de inclinación se calculan con las relaciones

\begin{displaymath} \vert u\vert=R=\sqrt{u \cdot u}=\sqrt{x^2+y^2} \end{displaymath}

(1)

y

\begin{displaymath} \alpha = \arctan \frac{y}{x}. \end{displaymath}

(2)

Page 48: Diseño Instruccional Matemáticas. “Trigonometría y ...conacyt.upemor.edu.mx/informe3/archivos/PropiedadInt...Diseño Instruccional Matemáticas.“Trigonometría y Geometría

48

El valor de R está definido sin ambigüedad, es decir dado el punto su valor es único. Sin

embargo, el ángulo de inclinación si puede ser ambiguo porque un ángulo siempre se mide

relativo a una recta de referencia. En el caso de la figura, la recta es el eje x positivo y el ángulo

esta medido en grados. Esta elección es la que parece más natural, sin embargo presenta un

detalle técnico relacionado con los signos, pues un punto en los cuadrantes donde las

coordenadas son mixtas (positivas y negativas) la función "\arctan" (arco tangente o tangente

inversa) regresa valores de ángulos negativos. Para recuperar el ángulo entre 0^\circ y

360^\circ, que equivale a entre 0 y 2\pi radianes se debe aplicar la corrección siguiente (para

más detalle checar esta página):

\begin{displaymath}\alpha=\left \lbrace \begin{array}{lll}\arctan (\frac{y}{x}) &amp;

\mathrm{si} &amp; x&gt;0,\ y\geq 0 \\ \arctan (\frac{y}{x}) +2\pi &amp; \mathrm{si} &amp;

x&gt;0,\ y &lt; 0 \\ \arctan (\frac{y}{x}) +\pi &amp; \mathrm{si} &amp; x&lt;0 \\ \frac{\pi}{2}

&amp; \mathrm{si} &amp; x=0 ,\ y &gt;0\\ \frac{3\pi}{2} &amp; \mathrm{si} &amp; x=0,\ y

&lt; 0 \end{array}\rigth.\end{displaymath},

donde asumimos que el resultado se desea radianes lo cual suele ser más cómodo para hacer

cálculos, pero si se desea en grados simplemente se usa así:

\begin{displaymath}\alpha=\left \lbrace \begin{array}{lll}\arctan (\frac{y}{x}) &amp;

\mathrm{si} &amp; x&gt;0,\ y\geq 0 \\ \arctan (\frac{y}{x}) +360^\circ &amp; \mathrm{si}

&amp; x&gt;0,\ y &lt; 0 \\ \arctan (\frac{y}{x}) +180^\circ &amp; \mathrm{si} &amp; x&lt;0 \\

90^\circ &amp; \mathrm{si} &amp; x=0 ,\ y &gt;0\\ 270^\circ &amp; \mathrm{si} &amp; x=0,\

y &lt; 0 \end{array}\rigth.\end{displaymath}.

En el ejemplo de la figura el vector u=(6,4) tiene un tamaño \vert u\vert=7.21 y

\alpha=33.69^\circ, estos dos valores corresponden a lo que entendemos por sus

``coordenadas polares''. Se puede mover el punto B con el ratón para crear diferentes vectores

y observar los ángulos y los tamaños que definen sus coordenadas polares.

Page 49: Diseño Instruccional Matemáticas. “Trigonometría y ...conacyt.upemor.edu.mx/informe3/archivos/PropiedadInt...Diseño Instruccional Matemáticas.“Trigonometría y Geometría

49

Si se tiene un vector definido en coordenadas polares, cabe la pregunta ¿cómo obtengo sus

coordenadas cartesianas x, y? En el ejemplo mencionado podemos ver que el punto B forma

un rectángulo dividido en dos por el vector u. Los triángulos que resultan de esta división son

iguales, pero reflejados respecto al vector, sin importar donde este colocado B (prueben

moverlo e intenten observar esta propiedad). Cualquier triángulo entonces tiene como catetos

a las coordenadas x e y y como hipotenusa a \vert u\vert=R, entonces se relacionan con el

ángulo \alpha mediante alguna función trigonométrica. En especial el seno y el coseno, de

modo que identificando a x como el cateto adyacente a \alpha y a y como el opuesto, sucede

que

\begin{displaymath} x=R \cos \alpha \end{displaymath}

(3)

y

\begin{displaymath} y=R \sin \alpha. \end{displaymath}

(4)

Usar coordenadas polares puede representar ventajas para algunos casos particulares, sin

embargo siempre hay un costo que pagar, por ejemplo, la suma en estas coordenadas no es la

suma de los tamaños y la suma de los ángulos. Hacer una suma implica regresar a las

componentes cartesianas y aplicar la definición de suma por componentes para al final volver

a coordenadas polares. Esto se hace comúnmente en física, por ejemplo cuando se suman

vectores de fuerza donde se conoce la magnitud de la fuerza y el ángulo al cual se aplica.

Sin embargo así como la suma se complica, el producto por escalar y el producto punto entre

vectores se simplifican, en estas coordenadas si un vector se escribe u_{\alpha}=(R,\alpha)

entonces al multiplicarlo por un escalar t no hay cambio en su dirección, solo en su tamaño,

por tanto

\begin{displaymath} t u_\alpha=(t R,\alpha). \end{displaymath}

(5)

De aquí se puede deducir que un vector cuya componente radial sea negativa, se genera

multiplicando otro vector por t=-1, es decir un signo simplemente indica un cambio en la

dirección del vector. Este cambio también podría interpretarse como un giro de 180^\circ en

el vector, el cual no cambia la dirección. De modo que (-R, \alpha)=(R,\alpha+180^\circ) si

0&lt;\alpha&lt;180^\circ y (-R, \alpha)=(R,\alpha-180^\circ) si

180^\circ&lt;\alpha&lt;360^\circ.

Si tenemos dos vectores en coordenadas polares u_\alpha=(R,\alpha) y v_{\alpha'}=(R',\alpha')

entonces usando la relación (ver lección de vectores)

Page 50: Diseño Instruccional Matemáticas. “Trigonometría y ...conacyt.upemor.edu.mx/informe3/archivos/PropiedadInt...Diseño Instruccional Matemáticas.“Trigonometría y Geometría

50

\begin{displaymath} u_\alpha \cdot v_{\alpha'} = \vert u_\alpha \vert\vert v_{\alpha'}

\vert\cos \theta, \end{displaymath}

(6)

donde \theta es el ángulo entre ambos vectores, es decir \theta=\alpha-\alpha', con lo que se

tiene que

\begin{displaymath} u_\alpha \cdot v_{\alpha'} = R R' cos(\alpha-\alpha'). \end{displaymath}

(7)

Noten que en las últimas ecuaciones utilizamos la notación u_\alpha, interpretándola como el

vector u escrito en coordenadas polares.

Estos son las nociones básicas para trabajar con ecuaciones en coordenadas polares. Ahora

podemos pasar a repasar algunos ejemplos.

2. Ecuaciones en coordenadas polares

Un vector en coordenadas polares u_\alpha=(R,\alpha) describe un punto en el plano. Si el

valor de R se hace variar en función del ángulo \alpha, se obtiene un conjunto de vectores que

dibujan ahora un lugar geométrico. Para comprender este concepto mejor tomemos un

ejemplo, imaginemos que R=1+\alpha (midiendo \alpha en radianes). Al variar \alpha entre 0 y

2\pi el vector u_\alpha va dibujando la figura que se muestra en la construcción siguiente

(para construirla presiona el botón de inicio de animación, para que varíe los valores de

\alpha):

Page 51: Diseño Instruccional Matemáticas. “Trigonometría y ...conacyt.upemor.edu.mx/informe3/archivos/PropiedadInt...Diseño Instruccional Matemáticas.“Trigonometría y Geometría

51

la figura se trata de una porción de una espiral. Cambia los parámetros k y a en la figura y nota

como la espiral cambia sus proporciones. Es importante mencionar cómo se hizo la

construcción en geogebra. En ésta versión del programa no hay (todavía) una herramienta

para graficar ecuaciones polares, sin embargo, siempre es posible dibujar un vector en

coordenadas cartesianas. El vector u en coordenadas cartesianas que corresponde al vector

u_\alpha=(R(\alpha),\alpha) es simplemente:

u=(R(\alpha) \cos \alpha,R(\alpha) \sin \alpha),

que para el caso de la espiral queda descrito así:

u=( (a+ k \alpha)\cos \alpha,(a+ k \alpha) \sin \alpha),

este vector si se puede dibujar directamente utilizando un "deslizador" que varíe \alpha. En el

ejemplo se usaron también deslizadores para cambiar variar los parámetros a y k.

Cabe mencionar aquí que siempre que tengamos una ecuación para R(\alpha) es posible

obtener un lugar geométrico. De hecho la elección puede dar resultados espectaculares.

Tomemos ahora por ejemplo

R(\alpha)=a \cos (k \alpha).

Noten que R puede tomar valores positivos o negativos, lo cual suena absurdo siendo que R

representa el tamaño del vector, sin embargo si hacemos consciente que el cambio de signo

solo representa un cambio de dirección y nos permitimos esa libertad, el dibujo que resulta

tomando a=2 y k=4 queda así (presiona nuevamente el botón para iniciar la animación o

mueve el deslizador de \alpha):

Page 52: Diseño Instruccional Matemáticas. “Trigonometría y ...conacyt.upemor.edu.mx/informe3/archivos/PropiedadInt...Diseño Instruccional Matemáticas.“Trigonometría y Geometría

52

la figura se llama Rosa Polar, y es diferente para valores distintos de a y k, prueba a

cambiarlos. El vector en coordenadas cartesianas que permite dibujar esta figura se escribe

así:

u=(a \cos (k \alpha) \cos \alpha,a \cos (k \alpha) \sin \alpha).

La pregunta en este punto es como se describen las cónicas con este tipo de ecuaciones, lo

cual se discutirá en la siguiente sección.

Ecuaciones polares de las cónicas.

Empecemos planteando un problema en forma de lugar geométrico: Encontrar la ecuación en

coordenadas polares de la elipse que tiene los focos en F1=(0,0) y F2=(4,0) y tiene un vértice

en V=(-2,0).

La solución es como sigue: El problema lo que implica es encontrar la función R(\alpha) que

describa a la elipse dada. Esta elipse se ve como lo muestra la figura siguiente. Ahí, el vector u

describe un punto arbitrario sobre la recta (recuerda que arbitrario quiere decir que lo puedes

mover con el ratón), pero como un foco está en (0,0) es también el vector que une el foco F1

con el punto. El vector w se escogió como el vector que va de F2 al mismo punto arbitrario y el

vector p apunta del origen (o F1) F2, es horizontal y su tamaño es la distancia focal f, en este

caso \vert p\vert=f=4.

La ecuación vectorial de la elipse nos dice que:

\begin{displaymath} \vert u\vert+\vert w\vert=S, \end{displaymath}

(8)

Page 53: Diseño Instruccional Matemáticas. “Trigonometría y ...conacyt.upemor.edu.mx/informe3/archivos/PropiedadInt...Diseño Instruccional Matemáticas.“Trigonometría y Geometría

53

donde S es la suma de las distancias de cualquier punto a los focos, en particular del vértice V.

De la figura, la distancia de V a F1 es 2 y de V a F2 vale 6, por tanto S=2+6=8, que como vimos

es el tamaño del eje mayor A. Por otro lado, también podemos observar que el vector u tiene

coordenadas polares u=(R,\alpha) por lo que \vert u\vert=R y además se cumple que w=u-p,

por tanto la ecuación [*] queda así:

\begin{displaymath} R+\vert u-p\vert=2a, \end{displaymath}

(9)

o bien así:

\begin{displaymath} \vert u-p\vert=A-R, \end{displaymath}

(10)

que implica que:

\begin{displaymath} \vert u-p\vert^2=(A-R)^2. \end{displaymath}

(11)

Pero como

\begin{displaymath} \vert u-p\vert^2=(u-p) \cdot (u-p)= \vert u\vert^2+\vert p \vert ^2-2\vert

u\vert\vert p\vert\cos \alpha=R^2+f^2-2R f\cos \alpha, \end{displaymath}

(12)

la ecuación equivale a

\begin{displaymath} R^2+f^2-2 R f\cos \alpha=(A-R)^2. \end{displaymath}

(13)

Finalmente, desarrollando el binomio del lado derecho simplificando y despejando R queda:

\begin{displaymath} R=\frac{(A^2+f^2)/A}{1-\frac{f}{A}\cos \alpha }. \end{displaymath}

(14)

Que sustituyendo los valores mencionados A=8 y f=4 queda:

\begin{displaymath} R=\frac{3}{1-\frac{1}{2}\cos \alpha }. \end{displaymath}

(15)

En general, toda elipse con eje mayor horizontal y un foco en el origen estará representada

por una ecuación como esta de modo que:

Page 54: Diseño Instruccional Matemáticas. “Trigonometría y ...conacyt.upemor.edu.mx/informe3/archivos/PropiedadInt...Diseño Instruccional Matemáticas.“Trigonometría y Geometría

54

\begin{displaymath} R=\frac{l}{1-e \cos \alpha}, \end{displaymath}

(16)

donde ahora l y e son dos constantes que dependen de su forma. El valor de l se puede

interpretar fácilmente, haciendo \alpha=90^\circ en la ecuación anterior. Esto corresponde a

tener un punto justo sobre el origen (prueben a hacerlo en la construcción de ejemplo). El

radio que define a este punto vale R=l que podemos recordar corresponde a la mitad del lado

recto de la elipse. Si L es el tamaño del lado recto l=L/2. Por otro lado e=\frac{f}{A} y es tan

importante que se le llama excentricidad y por tanto lo discutiremos en una sección especial.

Para tener más información sobre las ecuaciones polares de las cónicas restantes continúa con

los ejercicios de ejemplo de esta sección.

3. Sobre la excentricidad

De los problemas planteados y resueltos en la sección anterior se pueden inferir las siguientes

conclusiones:

1. Todas las ecuaciones polares de las cónicas cumplen una relación como esta:

\begin{displaymath} R=\frac{l}{1 \pm e \cos \alpha}. \end{displaymath}

(39)

Dónde l es la mitad del lado recto de la figura, e se define como excentricidad, que

corresponde con el cociente de la distancia focal entre la distancia entre vértices y el signo

\pm indica el origen esta en el foco izquierdo o derecho respectivamente.

2. Si en esta relación se toma e=0 se obtiene una circunferencia.

3. Si en esta relación se toma e=1 se obtiene una parábola.

4. Si los valores de e son mayores que 1 se dibujan hipérbolas.

5. Si los valores de e son menores que 1 se dibujan elipses.

Lo verdaderamente importante es que existe una ecuación general para las cónicas en

coordenadas polares muy simple y que depende del lado recto y la excentricidad, esta última

es igual al cociente entre la distancia focal F y la distancia entre los vértices A, de modo que:

\begin{displaymath} e=\frac{F}{A}. \end{displaymath}

(40)

Así en la circunferencia F=0 porque hay solo un foco. En la parábola solo hay un foco pero se

comporta como una elipse donde el segundo foco se separo una distancia muy grande, de

Page 55: Diseño Instruccional Matemáticas. “Trigonometría y ...conacyt.upemor.edu.mx/informe3/archivos/PropiedadInt...Diseño Instruccional Matemáticas.“Trigonometría y Geometría

55

hecho infinita. En esa circunstancia el cociente de F y A debe ser 1. Finalmente, en una

hipérbola siempre la distancia focal es mayor a la distancia entre sus vértices y en la elipse

esta relación se invierte.

Revisemos ahora el siguiente problema y su solución: encontrar la ecuación polar del lugar

geométrico de los puntos cuya distancia al origen es proporcional a la distancia a la recta x=p.

Solución: El lugar geométrico buscado suena como la definición de una parábola salvo por la

palabra ``proporcional''. Si tomamos u como el vector que apunta a un punto arbitrario del

lugar geométrico buscado, la ecuación vectorial que debe describir este lugar geométrico se

traduce así:

\begin{displaymath} \vert u\vert=k(p-\vert u\vert\cos \alpha), \end{displaymath}

(41)

donde k es una constante de proporcionalidad y \alpha es el ángulo respecto al eje x. Si \vert

u\vert=R podemos sustituir y despejar de modo que:

\begin{displaymath} R=\frac{pk}{1+k \cos \alpha}. \end{displaymath}

(42)

Dependiendo de los valores de p y k esta ecuación describe alguna cónica, por lo que el lado

recto es l=pk y el valor absoluto de k coincide con la excentricidad, es decir \vert k\vert=e.

Prueba la siguiente animación, ahí se puede controlar con un deslizador el valor de k y con el

otro el ángulo \alpha. Selecciona el valor de k y prueba iniciar la simulación. También se puede

mover el punto P por el que pasa la directriz. Con los criterios de la excentricidad dados

anteriormente se pueden obtener todas las cónicas. Solo se queda una pregunta de reflexión

que dejaremos abierta ¿Cómo se puede dibujar un circulo en la construcción?

Resumiendo, la excentricidad de un lugar geométrico definido por un foco fijo y una directriz

se define como el conciente entre la distancia de cualquier punto al foco y la distancia del

mismo punto a la directriz. Es deciír:

Page 56: Diseño Instruccional Matemáticas. “Trigonometría y ...conacyt.upemor.edu.mx/informe3/archivos/PropiedadInt...Diseño Instruccional Matemáticas.“Trigonometría y Geometría

56

\begin{displaymath} e=\frac{distancia[u,F]}{distancia[u,D]}, \end{displaymath}

(43)

con u representando al punto F al foco y D a la recta directriz. Curiosamente en las cónicas

este cociente coincide con el cociente de la distancia focal entre la distancia entre vértices, tal

y como se destacó en la sección anterior.

Como conclusión podemos decir que todas las cónicas pueden generalizarse en una sola

descripción como lugar geométrico y en una ecuación general, donde solo es necesario

conocer dos parámetros: el lado recto y la excentricidad, de ahí su importancia.

4. Ejercicios de Aplicación

5. Cuestionario

Módulo VI. Tangentes a cónicas

En esta sección abordaremos un problema que en su momento constituyó un reto fundamental de

geometría analítica y que también fue una motivación que llevó al desarrollo del cálculo

diferencial: el problema de las tangentes a curvas planas. Los objetivos, para este caso, se pueden

resumir en los siguientes puntos:

1. Recordar los métodos analíticos para resolver el problema de tangentes a cónicas.

2. Valorar lo que llamaremos el Método Diferencial para obtener tangentes a curvas.

Por tanto esta lección se dividirá en dos secciones principales, cada una con un conjunto de

ejercicios. Recomendamos empezar por la sección “Secantes y tangentes” al menos la primera vez

que se realice la sección.

1. Secantes y tangentes

En general cuando se tiene un conjunto de puntos que describen un lugar geométrico, éste

puede interceptarse en uno o varios puntos con una recta dada. En particular si se trata de

cónicas, una recta solo puede hacer intersección con ellas en uno, dos o ningún punto. De

entrada se podría demostrar que es imposible que se cruce en más de dos. Cuando una recta

tiene dos intersecciones la bautizamos (o definimos) con el nombre de “secante” a la curva. Si

hay solo una intersección se trata de una “tangente”. En la siguiente figura, se pueden

observar las secantes y las tangentes a una cónica. Presionando el botón para iniciar

animación se puede observar como una recta de pendiente m y ordenada al origen b puede o

no hacer intersección a la cónica. La animación cambia de manera continua el valor de b pero

con los deslizadores pueden cambiarse también la pendiente de la recta y los parámetros de la

cónica. Variando la excentricidad representada con el valor e se pude obtener cualquier cónica

con lado recto igual al doble del valor del deslizador l. Los puntos de intersección están

definidos por D y C y se nota como pueden estar o no definidos.

Page 57: Diseño Instruccional Matemáticas. “Trigonometría y ...conacyt.upemor.edu.mx/informe3/archivos/PropiedadInt...Diseño Instruccional Matemáticas.“Trigonometría y Geometría

57

La solución gráfica que se muestra en geogebra tiene una contraparte algebraica que puede

seguirse de la siguiente manera:

* Tomemos la ecuación de una cónica centrada en un foco, con excentricidad e y semilado

recto l. En coordenadas polares es:

R=\frac{l}{1-e \cos \alpha}.

* Usando que R=\sqrt{x^2+y^2} y x=R \cos \alpha, en la ecuación anterior obtenemos la

ecuación cartesiana:

(1-e^2)x^2-2lex+y^2=l^2.

Que si ponemos a=(1-e^2), b=-2le y c=1 y d=-l^2 tiene la forma general

a x^2 +b x+c y^2 +d=0.

* Para que la recta y=mx+b intersecte a esta cónica es necesario que las dos ecuaciones

tengan soluciones simultáneamente. Para hacerlo se puede sustituir y=mx+b en la cónica y

resolver la ecuación de segundo grado para x:

(b^2 + d )+ (b + 2 b m) x + (a + m^2) x^2=0

* La ecuación de segundo grado se puede resolver de manera general para x quedando:

x_\pm=\frac{-(b+2 b m)\pm\sqrt{(b+2 b m)^2-4 \left(a+m^2\right) \left(b^2+d\right)}}{2

\left(a+m^2\right)}.

* Las coordenadas y de los puntos de intersección son y_\pm=m x_\pm +b.

Noten que si el discriminante es menor que cero la raíz en la solución no se puede calcular lo

que indica que no hay intersección y si el discriminante se hace cero tenemos una tangente.

De este modo una recta tangente a esta cónica centrada en un foco debe cumplir que

Page 58: Diseño Instruccional Matemáticas. “Trigonometría y ...conacyt.upemor.edu.mx/informe3/archivos/PropiedadInt...Diseño Instruccional Matemáticas.“Trigonometría y Geometría

58

(b+2 b m)^2-4 \left(a+m^2\right)\left(b^2+d\right)=0.

Este ejemplo permite ayudar a plantear y resolver el problema importante de esta sección,

que de alguna manera es al revés del anterior, pues dado un punto sobre una cónica se trata

de encontrar la ecuación de la recta que pasa por él y es tangente a la curva. Así, para resolver

el problema hay que suponer que e y l están dadas y se tiene que determinar m y b en un

punto seleccionado (x_0,y_0). Ilustremos la solución con un ejemplo.

Supongamos que e=\frac{1}{2} y l=2, es decir la cónica es una elipse con ecuación:

\frac{3}{4}x^2-2x+y^2=4,

y se desea encontrar la tangente en el punto A=(0,2). Lo primero es verificar que el punto esta

en la cónica, lo cual se hace sustituyendo en la ecuación y verificando la igualdad, lo cual es

cierto. Como el punto está en la recta tangente y=mx+b se cumple que

2= m(0)+b.

Por lo que b=2. Entonces todos los puntos de la recta cumplen que y=mx+2. Para encontrar la

intersección de esta recta con la cónica sustituimos y en su ecuación por lo que:

\frac{3}{4}x^2-2x+(m x+2)^2=4,

de modo que, desarrollando y simplificando queda la ecuación:

(m^2 +\frac{3 }{4})x^2+(4 m -2)x =0.

Como en el ejemplo anterior esta ecuación tiene una sola solución solo si su discriminante vale

cero, es decir si:

(4m-2)^2-4(m^2 +\frac{3 }{4})(0)=0

o bien:

(4m-2)^2=0.

lo que se cumple solo si m=\frac{1}{2} de modo que la tangente debe tener por ecuación

y=\frac{1}{2}x+2,

Este procedimiento es general para resolver el problema y se puede resumir de la siguiente

manera:

1. Con el punto donde pasa la tangente se determina la ordenada al origen de la recta

tangente en función de su pendiente.

2. Se sustituye la ecuación de la recta parcialmente conocida en la ecuación de la cónica.

3. Se plantea la ecuación de segundo grado que resuelve las intersecciones.

Page 59: Diseño Instruccional Matemáticas. “Trigonometría y ...conacyt.upemor.edu.mx/informe3/archivos/PropiedadInt...Diseño Instruccional Matemáticas.“Trigonometría y Geometría

59

4. Se calcula el discriminante de esa ecuación y se iguala a cero; de ahí se despeja la pendiente

de la recta tangente.

2. Método diferencial

Empezamos ahora con un ejemplo. Tenemos una elipse cuya ecuación cartesiana es:

x^2+3y^2=1.

Esta se muestra en la construcción siguiente:

A la construcción se agregaron los puntos C y D sobre la elipse, las coordenadas en el eje x de

estos dos están representadas por los puntos A y B y su diferencia es h. Las coordenadas en el

eje y, de estos mismos, los representan los puntos F y E y su diferencia vale k. Se agregó un

deslizador con una variable h que controla la posición de B, es decir la coordenada x del punto

D. Al mover el deslizador se puede notar como decreciendo h el punto D se aproxima a C. Se

trazaron también la recta secante que pasa por C y D (nombrada d) y la tangente a la elipse en

C (nombrada g). Noten que al decrecer h al mínimo la recta d se aproxima a g tanto como se

quiera. Es decir cuando h tiende a cero (sin ser cero) la pendientes de ambas rectas se

aproximan mucho hasta el punto de ser iguales. Esto induce a pensar un método para

determinar la pendiente de la recta tangente a partir de la recta secante.

Si generalizamos el punto C a C=(x_0,y_0) entonces el punto D estaría dado por

D=(x_0+h,y_0+k). De la figura se puede ver que la pendiente de la recta secante es:

m_s=\frac{k}{h} por lo que D=(x_0+h,y_0+m_s h).

Como C está en la elipse, entonces cumple su ecuación:

x_0^2+3y_0^2=1. (1)

Como D también está en la elipse, entonces también cumple su ecuación:

(x_0+h)^2+3(y_0+m_s h)^2=1. (2)

Page 60: Diseño Instruccional Matemáticas. “Trigonometría y ...conacyt.upemor.edu.mx/informe3/archivos/PropiedadInt...Diseño Instruccional Matemáticas.“Trigonometría y Geometría

60

Si ahora desarrollo los cuadrados perfectos:

x_0^2 +2 h x_0 +h^2+3(y_0^2+2 m_s h y_0+m_s^2 h^2)=1,

por lo que distribuyendo el 3 y arreglando los términos

(x_0^2+3y_0^2)+ 2 h x_0+6 m_s h y_0+h^2+m_s^2 h^2=1.

Pero noten que lo que dejamos entre paréntesis es la ecuación para el punto C por lo que se

puede sustituir por un 1, que a su vez se elimina con el miembro derecho de la ecuación, por

tanto:

2 h x_0+6 m_s h y_0+h^2+m_s^2 h^2=0.

Finalmente factorizando una h y eliminándola, queda que:

2 x_0+6 m_s y_0+h+m_s^2 h=0.

Si ahora tomamos h=0, lo que equivale a juntar los dos puntos queda simplemente que:

2 x_0+6 m_s y_0=0, (3)

de donde se puede despejar m_s, de modo que:

m_s=-\frac{x_0}{3 y_0}.

Pero al haber hecho h=0 la secante coincide con la tangente por lo que m_s es también la

pendiente de la tangente m_t. En especial en el dibujo C=(\frac{1}{2},\frac{1}{2}) de modo que

m_t=-\frac{1}{3}, lo cual puede verificarse gráficamente.

Aquí hay que resaltar dos puntos, al desarrollar la ecuación (2), encontramos información

escondida en ella, si ahí se toma h=0 no permite obtener el valor de m_s. Es el desarrollo

algebraico el que libera la información. Así para resolver el problema exactamente es

necesario desarrollar las ecuaciones y luego tomar el valor de h=0. Este proceso coincide con

lo que se conoce por el concepto de límite y en particular el proceso resultante se llama

derivación.

Para aquellos que saben derivar noten que la ecuación de la elipse era:

x^2+3y^2=1

si derivo respecto a x cada término:

2 x +6 y \frac{dy}{dx}=0,

y despejo \frac{dy}{dx}:

\frac{dy}{dx}=-\frac{x}{3y}.

Page 61: Diseño Instruccional Matemáticas. “Trigonometría y ...conacyt.upemor.edu.mx/informe3/archivos/PropiedadInt...Diseño Instruccional Matemáticas.“Trigonometría y Geometría

61

Por tanto la pendiente de la tangente al punto (x_0,y_0) es m_t=\frac{dy}{dx}=-

\frac{x_0}{3y_0}, es decir es la derivada de la variable y en función de x evaluada en el punto

dado.

Si sabemos derivar encontrar las pendientes de las tangentes es directo. De hecho el cálculo

diferencial se construyó para formalizar este método de obtención de tangentes, el cual se

cómo se puede notar resuelve un problema algebraico laborioso en tres pasos.

En esta liga están las tablas con las derivadas de las funciones básicas. Para un maestro de

geometría analítica dominar el cálculo diferencial es un arma inapreciable en la solución de

problemas con un nivel de complejidad mayor. Este curso no pretende extenderse mas, sin

embargo espero que sea un estimulo para recordar y ampliar por cuenta propia los conceptos

del cálculo diferencial.

3. Ejercicios

4. Cuestionario

4.1. Contenido Adicional. Ejercicios de Entrenamiento

Examen Final

Page 62: Diseño Instruccional Matemáticas. “Trigonometría y ...conacyt.upemor.edu.mx/informe3/archivos/PropiedadInt...Diseño Instruccional Matemáticas.“Trigonometría y Geometría

62

Banco de preguntas

Vectores

1. Tenemos los vectores u=(3,5), w=(-2,1) y s=(0,6). Encuentra un vector v para cual la

ecuación w+s=v-u se cumple.

a. $$v\=(1,12)$$

b. $$v\=(-5,2)$$

c. $$v\=(0,7)$$

2. Determine si los vectores (-3,7), (6,-14) son paralelos perpendiculares o bien oblicuos.

a. Paralelos

b. Perpendiculares

c. Oblicuos

3. ¿Cuál es el par ordenado (x,y) tal que la flecha que va de A=(-1,-2) a B=(x,y) representa el

mismo vector que el vector que va de C=(2,4) a D=(8,-2)?

a. $$B\=(5,-8)$$

b. $$B\=(-5,8)$$

c. $$B\=(7,-4)$$

4. ¿Cuál es el valor de y que hace al vector v=(-4,y) ser perpendicular a w=(3,2)?

a. $$y\=6$$

b. $$y\=3$$

c. $$y\=0$$

5. Calcule el ángulo que se forma entre los vectores v=(-4,3) y w=(1,0). Elige la opción

correcta.

a. el ángulo es 2.49 radianes

b. el ángulo es 110 grados

c. el ángulo es 0 grados

6. Elige de las opciones que se presentan aquella que tenga un vector unitario (es decir de

tamaño 1) que apunte en la misma dirección del vector u=(1,3).

a. $$(\frac\{1\}\{\sqrt\{10\}\},\frac\{3\}\{\sqrt\{10\}\})$$

b. $$(\frac\{1\}\{\sqrt\{4\}\},\frac\{3\}\{\sqrt\{4\}\})$$

c. $$(\frac\{1\}\{\sqrt\{2\}\},\frac\{1\}\{\sqrt\{2\}\})$$

7. Si definimos los vectores u=(3,1), v=(-1,-3), w=(-2,1), s=(1,-1) y r=(w+s) \cdot s ( w-

\frac{1}{2}v ); calcula y captura en el espacio de "respuesta" el valor del tamaño de r.

a. 2.92

8. ¿Cuánto vale la proyección del vector v=(-2,3) sobre el vector u=(3,1)?

a. -0.94

9. Tenemos dos vectores u=(2,1) y v=(-1,2) que representan dos lados del triángulo con

vértices en (0,0), (2,1) y (-1,2). Si w es un vector que representa al tercero de sus lados,

calcula el valor absoluto de la proyección de w sobre v y da entrada a la respuesta.

Page 63: Diseño Instruccional Matemáticas. “Trigonometría y ...conacyt.upemor.edu.mx/informe3/archivos/PropiedadInt...Diseño Instruccional Matemáticas.“Trigonometría y Geometría

63

a. 2.24

10. Dados los vectores u=(2, k) y v= (3, - 2), calcula el valor de k para que u y v sean

perpendiculares.

a. 3

11. Si tenemos dos vectores u=(1,0) y v=(0,9) ¿cuánto vale la proyección de u sobre v?

a. 0

12. ¿Cuál es el par ordenado (x,y) tal que el vector que va de A=(x,y) a B=(6,-2) representa el

mismo vector que la flecha que va de C=(4,-7) a D=(3,1)?

a. $$A\=(7,-10)$$

b. $$A\=(5,6)$$

c. $$A\=(1,-4)$$

Lugares Geométricos I

1. Selecciona la ecuación del lugar geométrico de los puntos P=(x,y) tales que la pendiente

del segmento que conecta a cada punto P con S=(1,-3) es el doble de la pendiente del

segmento que lo conecta con T=(3,2)

a. $$y\=\frac\{7x-13\}\{x+1\}$$

b. $$y\=\frac\{1\}\{2\}(5x-11)$$

c. $$y\=\frac\{1\}\{2\}(5x+11)$$

d. $$y\=-\frac\{8x-20\}\{x-5\}$$

2. Selecciona la ecuación del lugar geométrico de los puntos P=(x,y) tales que la pendiente

del segmento de recta que conecta cada punto P conS=(1,-2) es 3 unidades menor que la

pendiente del segmento de recta que conecta al punto con T=(-1,2).

a. $$y\=\frac\{1\}\{2\}(3-4x-3x^2)$$

b. $$y\=\frac\{1\}\{2\}(-3-4x-3x^2)$$

c. $$y\=\frac\{4x-2\}\{x-2\}$$

3. Se tiene un lugar geométrico descrito por los puntos P tales que la pendiente del

segmento que va de P a un punto A es cuatro veces la pendiente de P a un punto B. Si u, v

y w son tres vectores que van del origen a los puntos P, A y B respectivamente y se utilizan

los vectores unitarios i=(1,0) y j=(0,1), deduce y selecciona la ecuación vectorial que

describe al lugar geométrico.

a. $$\frac\{(u-v) \cdot j\}\{(u-v)\cdot i\}\=4 \frac\{(u-w) \cdot j\}\{(u-w)\cdot

i\}$$

b. $$ 4 \frac\{(u-v) \cdot j\}\{(u-v)\cdot i\}\= \frac\{(u-w) \cdot j\}\{(u-w)\cdot i\}$$

c. $$\frac\{(u-v) \cdot i\}\{(u-v)\cdot j\}\=4 \frac\{(u-w) \cdot i\}\{(u-w)\cdot j\}$$

4. Se tienen dos circunferencias iguales de radio r=3. El centro de la primera se encuentra en

el punto C_1=(3,-2) y el de la segunda en C_2=(3,2). Selecciona de la opciones, el par de

puntos donde se intersectan ambas circunferencias.

a. $$(3-\sqrt\{5\},0)$$ y $$(3+\sqrt\{5\},0)$$

b. $$(-\sqrt\{5\},0)$$ y $$(\sqrt\{5\},0)$$

c. No se intersectan en ningún punto.

Page 64: Diseño Instruccional Matemáticas. “Trigonometría y ...conacyt.upemor.edu.mx/informe3/archivos/PropiedadInt...Diseño Instruccional Matemáticas.“Trigonometría y Geometría

64

d. $$(2-\sqrt\{5\},0)$$ y $$(2+\sqrt\{5\},0)$$

5. Una circunferencia está representada por la ecuación x^{2}+y^{2}-2x-4y-20=0. ¿Dónde

está colocado su centro y cuánto vale su radio?

a. Centro en $$(1,2)$$ y radio $$r\=5$$.

b. Centro en $$(-1,-2)$$ y radio $$r\=25$$.

c. Centro en $$(2,1)$$ y radio $$r\=5$$.

d. Centro en $$(2,-1)$$ y radio $$r\=25$$.

6. Un objeto gira alrededor del origen de un plano cartesiano de modo que a un tiempo t se

encuentra en la posición dada por el vector u=(3 \cos 2t, 3 \sin 2t). Elige entre las opciones

a la ecuación cartesiana de la figura que dibuja la trayectoria de este objeto.

a. $$x^2+y^2\=9$$

b. $$(x-2)^2+(y-2)^2\=9$$

c. $$x^2+y^2\=3$$

d. $$\frac\{x^2\}\{9\}+\frac\{y^2\}\{4\}\=1$$

7. Una elipse tiene una ecuación cartesiana 9x^2+4y^2-36=0. Cuál será la ecuación de una

circunferencia que tiene el mismo centro que esta elipse y se encuentra inscrito en ella.

a. $$x^2+y^2\=4$$

b. $$x^2+y^2\=2$$

c. $$x^2+y^2\=9$$

d. $$x^2+y^2\=36$$

8. ¿Cuál es la ecuación de la elipse que tiene sus focos en F_1=(-3.0) y F_2=(3,0) y tiene un

vértice en V_1=(0,2)?

a. $$\frac\{x^2\}\{13\}+\frac\{y^2\}\{4\}\=1$$

b. $$\frac\{x^2\}\{13\}+\frac\{x^2\}\{4\}\=1$$

c. $$\frac\{x^2\}\{2\}+\frac\{x^2\}\{\sqrt\{13\}\}\=1$$

d. $$ x^2+x^2\=26$$

9. ¿Cuánto vale el lado recto de la elipse descrita por la ecuación cartesiana \frac{(x-

1)^2}{4}+\frac{(y-1)^2}{9}=1?

a. 2.66

10. Una circunferencia pasa por tres puntos A=(1,0), B=(6,0) y C=(3,3). Usando geogebra,

encuentra el radio de la circunferencia que pasa por ellos y captura su valor en el espacio

de respuesta. Redondea a dos decimales tu resultado.

a. 2.55

Lugares Geométricos II

1. ¿Cuál es la ecuación de la parábola con vértice en el origen y foco F(0,5), y cuál es la

ecuación de su directriz?

a. Parábola $$y^2\=20x$$; directriz $$y\=-25$$

b. Parábola $$x^2\=20y$$; directriz $$y\=-25$$

c. Parábola $$x^2\=20y$$; directriz $$y\=-5$$

d. Parábola $$y^2\=20y$$; directriz $$x\=-5$$

Page 65: Diseño Instruccional Matemáticas. “Trigonometría y ...conacyt.upemor.edu.mx/informe3/archivos/PropiedadInt...Diseño Instruccional Matemáticas.“Trigonometría y Geometría

65

2. Encuentra la ecuación del círculo que pasa por los focos de las hipérbolas cuyas

ecuaciones son 9x^2-16 y^2=144 y 16y^2-9x^2=144. Dibuja las hipérbolas y el círculo.

Utiliza Geogebra para dibujar la ecuación y sube tu archivo aquí.

a. $$x^2 +y^2\=16$$

b. $$x^2 +y^2\=25$$

c. $$x^2+y^2\=9$$

d. $$x^2+y^2\=\sqrt\{52\}$$

3. Calcule las coordenadas del foco, obtenga una ecuación cartesiana de la directriz y de la

parábola con vértice en el origen y que pasa por los puntos S(3,3), T(3,-3).

a. Foco (1.25,0); Parábola $$y^2\=3x$$; directriz $$x\=-3$$

b. Foco (0, 1.25); Parábola $$x^2\=3y$$; directriz $$y\=1.25$$

c. Foco (1.25, 0); Parábola $$y^2\=3x$$ ; directriz $$y\=3$$

d. Foco (0, 0.75); Parábola $$y^2\=3x$$ ; directriz $$x\=-0.75$$

4. Emplea la definición de la parábola para obtener una ecuación cartesiana de la parábola

con foco F(5,2) y directriz x=1.

a. $$10x\=y^2+4y+28$$

b. $$10x-y^2+4y+28\=0$$

c. $$8x\=y^2-4y+28$$

d. $$8x\=y^2+4y+28$$

5. Obtenga la ecuación de la parábola que pasa por los punto dados Q=(0,2), S=(1.5,-1) y T=(-

0.5,1)

a. $$y\=-2x^2+x+2$$

b. $$y\=2x^2-x+2$$

c. $$y\=-2x^2+x-2$$

d. $$x\=-2y^2+x+2$$

6. Un arco parabólico, tiene diez metros de altura, con el eje vertical y cuyos puntos de

apoyo están separados por 20m. ¿Está el foco de la parábola sobre el suelo o debajo de él,

y a qué distancia de la superficie está?

a. El foco está a 5.0 metros por abajo del suelo

b. El foco está a 7.5 metros por arriba del suelo

c. El foco está sobre el suelo

d. El foco está a 7.5 metros por abajo del suelo

7. Se tiene un reflector parabólico cuya forma se obtiene haciendo girar un arco de parábola

que empieza en el vértice, alrededor del eje de la parábola. Si el foco está a 9cm del

vértice y el arco parabólico tiene 16cm de profundidad. ¿Cuál es la abertura del reflector?

a. 40cm

b. 30cm

c. 48cm

d. 20cm

8. Hay que encerrar a un campo rectangular mediante una cerca de 120m de longitud. Si el

área es y cuando uno de los lados tiene longitud x, entonces y=60x - x^2. Trace la gráfica

de esta ecuación para x en [0,60] y determine el valor de x para que el área sea máxima.

Page 66: Diseño Instruccional Matemáticas. “Trigonometría y ...conacyt.upemor.edu.mx/informe3/archivos/PropiedadInt...Diseño Instruccional Matemáticas.“Trigonometría y Geometría

66

a. $$x\=30m$$

b. $$x\=20$$

c. $$x\=25$$

d. $$x\=40$$

9. La hipérbola y la elipse cuyas ecuaciones son 3x^2-y^2=12 y

9x^2+25y^2=225,respectivamente, tienen los mismos focos cuyos valores son:

a. Los focos están en $$(-2,0)$$ y $$(2,0)$$

b. Los focos están en $$(-5,0)$$ y $$(5,0)$$

c. Los focos están en $$(-3,0)$$ y $$(3,0)$$

d. Los focos están en $$(-4,0)$$ y $$(4,0)$$

10. Dos observadores están en los puntos A(-5,0) y B(5,0), respectivamente. Un cañón se

encuentra en un lugar Q(x,6). El observador en A oye un disparo 18 segundos después del

momento en que lo oye el observador B. Encuentra la posición del cañón. Considera que

el sonido viaja a \frac{1}{3}km/s.

a. $$(3/4 \sqrt\{52\},6)$$ y $$ (-3/4 \sqrt\{52\},6)$$

b. $$ (3/4 \sqrt\{20\},6)$$ y $$ (-3/4 \sqrt\{20\},6)$$

c. $$ (3/4 \sqrt\{25\},6)$$ y $$ (-3/4 \sqrt\{25\},6)$$

d. $$ (3/4 \sqrt\{28\},6)$$ y $$ (-3/4 \sqrt\{28\},6)$$

Coordenadas polares

1. Si se tiene la siguiente ecuación polar, R=\frac{4}{4-8 \cos \alpha} ¿de qué curva se trata?

a. Parábola

b. Hipérbola

c. Elipse

d. Circunferencia

2. Elige el vector que corresponda a la suma de los vectores en coordenadas polares

u_\alpha=(2,30^\circ) y v_\alpha=(1,45^\circ).

a. $$(2.9772,34.98^\circ)$$

b. $$(8.8637,34.98^\circ)$$

c. $$(3,75^\circ)$$

d. 8.8637,14.987^\circ)$$

3. De las opciones siguientes elige la ecuación de una elipse.

a. $$R\=\frac\{6\}\{3+2 \cos \alpha\}$$

b. $$R\=\frac\{6\}\{3 \sin \alpha+2 \cos \alpha\}$$

c. $$R\=\frac\{6\}\{3 +6 \cos \alpha\}$$

d. $$R\=\frac\{1\}\{1 +6 \cos \alpha\}$$

4. Elige una ecuación polar que corresponda a la recta descrita por y=3x+1.

a. $$R\=\frac\{1\}\{\sin (t)-3 \cos (t)\}$$

b. $$R\=\frac\{1\}\{1-3 \cos (t)\}$$

c. $$R\=\frac\{1\}\{3 \sin (t)-3 \cos (t)\}$$

d. $$R\=3 \sin (t)-3 \cos (t)$$

Page 67: Diseño Instruccional Matemáticas. “Trigonometría y ...conacyt.upemor.edu.mx/informe3/archivos/PropiedadInt...Diseño Instruccional Matemáticas.“Trigonometría y Geometría

67

5. ¿Qué lugar geométrico representa la ecuación polar R=3\sin \alpha -3 \cos \alpha?

a. Una circunferencia con centro fuera del origen.

b. Una recta

c. Una elipse con vértice en el origen.

d. Una elipse centrada en el origen.

6. Un vector en coordenadas polares esta descrito por u_\alpha=(3,120^\circ). Elige su

representación en coordenadas cartesianas

a. $$u\=(-\frac\{3\}\{2\},\frac\{3\sqrt\{3\}\}\{2\})$$

b. $$u\=(\frac\{1\}\{2\},-\frac\{\sqrt\{3\}\}\{2\})$$

c. $$u\=(\frac\{3\}\{2\},-\frac\{\sqrt\{3\}\}\{2\})$$

d. $$u\=(-\frac\{3\}\{2\},\frac\{\sqrt\{2\}\}\{2\})$$

7. Elige entre las opciones la ecuación polar de una parábola con foco en (0,0) y vértice en (-

1,0).

a. $$R\=\frac\{2\}\{1-\cos \alpha\}$$

b. $$R\=\frac\{-2\}\{1-\cos \alpha\}$$

c. $$R\=\frac\{1\}\{1-2 \cos \alpha\}$$

d. $$R\=\frac\{1\}\{1- \cos \alpha\}$$

8. La ecuación polar de una parábola está dada por: R=\frac{6}{3-3 \cos \alpha}, ¿Cuánto vale

su lado recto?

a. 4

9. Encuentra y captura el valor de la distancia focal de la elipse descrita por la ecuación polar:

R=\frac{4}{1-0.5 \cos \alpha}

a. 5.33

10. La excentricidad de la órbita que describe la tierra alrededor del sol es e=0.01671. Si su

perihelio vale 0.983 UA (unidades astronómicas) ¿cuánto vale su afelio?

a. 1.01641

Tangentes acónicas

1. Una cónica tiene como ecuación 4y^2-2x^2-2x+6y+5=0. Selecciona, entre las opciones la

que corresponda a la derivada de esta ecuación.

a. $$(8y+6)\frac\{dy\}\{dx\}-4x-2\=0$$

b. $$8y\frac\{dy\}\{dx\}-4x-2\=0$$

c. $$6\frac\{dy\}\{dx\}-4x-2\=0$$

d. $$(8y+6)y-4x-2\=0$$

2. Una hipérbola tiene como ecuación x y=1. Elige la ecuación de su tangente en el punto

P=(1,1).

a. $$x+y\=2$$

b. $$x-y\=0$$

c. $$0.5x-0.5y\=0$$

d. $$0.5x+0.5y\=-1$$

Page 68: Diseño Instruccional Matemáticas. “Trigonometría y ...conacyt.upemor.edu.mx/informe3/archivos/PropiedadInt...Diseño Instruccional Matemáticas.“Trigonometría y Geometría

68

3. El punto de intersección (x,y) entre una recta y una cónica se puede resolver con la

ecuación m^2 x^2+2 m x+6 x-8=0. Elige entre las opciones la que contenga el

discriminante de esta ecuación para x.

a. $$12 (3 + 2 m + 3 m^2)$$

b. 12 (3 + 2 m )$$

c. $$3 + 2 m + 3 m^2$$

d. $$12$$

4. Elige la ecuación que corresponda a la recta tangente de la cónica x^{2}+3x+4y=0 en el

punto P=(0,0).

a. $$y\=-\frac\{3\}\{4\}x$$

b. $$y\=-\frac\{4\}\{3\}x$$

c. $$y\=\frac\{4\}\{3\}x$$

d. $$y\=\frac\{3\}\{4\}x$$

5. Elige las ecuaciones de las rectas que pasan por el punto S=(0,0) y son tangentes a la

parábola x=y^{2}+9.

a. $$y\=\frac\{1\}\{6\}x$$ e $$y\=-\frac\{1\}\{6\}x$$

b. $$y\=\frac\{1\}\{6\}x+2$$ e $$y\=-\frac\{1\}\{6\}x-2$$

c. $$x\=\frac\{1\}\{6\}y$$ e $$x\=-\frac\{1\}\{6\}y$$

d. $$x\=\frac\{1\}\{3\}y$$ e $$x\=-\frac\{1\}\{3\}y$$

6. Encuentra el punto en que se intersectan la recta y=6-x y la parábola y^2+6x-6y=0. Elige la

respuesta correcta.

a. $$(0,6)$$

b. $$(0,0)$$

c. $$(6,0)$$

d. $$(5/6,1)$$

7. Usando geogebra calcula en ángulo de intersección entre las curvas y^2+\frac{x^2}{9}=1 y

y^2+\frac{(x-3)^2}{9}=1. Captura el resultado, en grados, en el campo de respuesta.

a. 158.21

8. Captura el valor de la ordenada al origen b de la recta y=\frac{2}{3}x+b si esta es tangente

a la cónica x^{2}+4y^{2}-4x+6y=0.

a. 4.167

9. Captura el valor de la ordenada al origen b de la recta y=m x+b si esta es tangente a la

cónica y=4-x^2 y pasa por el punto P=(0,5).

a. 5

10. Captura el valor de la pendiente a la cónica x^{2}+y^{2}+3x+5y=0 en el punto P=(1,-4).

a. 1.667

Banco de preguntas Examen Diagnostico y Final

1. ¿A cuál de las siguientes figuras describe la ecuación x^2 /9 - y/4 = 1?:

Page 69: Diseño Instruccional Matemáticas. “Trigonometría y ...conacyt.upemor.edu.mx/informe3/archivos/PropiedadInt...Diseño Instruccional Matemáticas.“Trigonometría y Geometría

69

a.

b.

c.

d.

e. 2. ¿Cuál de las ecuaciones en la lista representa correctamente a la parábola de la figura

siguiente?:

Page 70: Diseño Instruccional Matemáticas. “Trigonometría y ...conacyt.upemor.edu.mx/informe3/archivos/PropiedadInt...Diseño Instruccional Matemáticas.“Trigonometría y Geometría

70

a. $$y\=x^2-4$$

b. $$y\=x^2+4$$

c. $$x\=-y^2+16$$

d. $$x\=y^2-16$$

e. $$y\=-x^2+4$$

3. ¿Cuánto vale la excentricidad e de la cónica representada por la ecuación 4 x^2+4 y^2 =3?

a. $$e\=1/2$$

b. $$e\=1$$

c. $$e\=0$$

d. $$e\=2$$

e. $$e\=-2$$

4. Determinar la ecuación, centro y radio de la circunferencia que pasa por los puntos (6,2),

(8,0), cuyo centro está sobre la recta 3x+7y+2=0:

Page 71: Diseño Instruccional Matemáticas. “Trigonometría y ...conacyt.upemor.edu.mx/informe3/archivos/PropiedadInt...Diseño Instruccional Matemáticas.“Trigonometría y Geometría

71

a. $$(x-4)^2+(y+2)^2\=20$$

b. $$(x-2)^2+(y+4)^2\=20$$

c. $$(x-4)^2+(y-2)^2\=20$$

d. $$\sqrt[n]\{(x-4)\}+\sqrt[n]\{(y+2)\}\=20$$

e. $$(x-4)^2+(y-2)^2\=20^2$$

5. Determinar la ecuación de la circunferencia circunscrita al triángulo cuyos vértices son:

P_1(-1,1), P_2(3,5) y P_3(5,-3):

a. $$(x-\frac\{16\}\{5\})^2+(y-\frac\{4\}\{5\})^2\=\frac\{442\}\{25\}$$

b. $$(x+\frac\{16\}\{5\})^2+(y+\frac\{4\}\{5\})^2\=\frac\{442\}\{25\}$$

c. $$(x-\frac\{16\}\{5\})+(y-\frac\{4\}\{5\})\=\frac\{442\}\{25\}$$

d. $$\sqrt[]\{(x-\frac\{16\}\{5\})^2\}+\sqrt[]\{(y-

\frac\{4\}\{5\})^2\}\=\frac\{442\}\{25\}$$

e. $$(x-\frac\{4\}\{5\})^2+(y-\frac\{16\}\{5\})^2\=\frac\{442\}\{25\}$$

6. Determine la ecuación de la elipse con vértices (\pm 4,0) y focos (\pm 2,0):

a. $$3x^2 + 4y^2 \= 48$$

b. $$2x^2 + 4y^2 \= 48$$

c. $$2x^2 - 4y^2 \= 48$$

d. $$4y^2 - 2x^2 \= 48$$

e. $$4x^2 + 2y^2 \= 48$$

7. Determine la ecuación de una hipérbola que tiene las siguientes características. Vértices

son los puntos V(0,3) y V'(0,-3). Focos F(0,5) y F'(0,-5):

a. $$\frac\{y^2\}\{9\}-\frac\{x^2\}\{16\}\=1$$

b. $$\frac\{y^2\}\{16\}-\frac\{x^2\}\{9\}\=1$$

c. $$\frac\{y^2\}\{3\}-\frac\{x^2\}\{4\}\=1$$

d. $$\frac\{y^2\}\{9\}+\frac\{x^2\}\{16\}\=1$$

e. $$\frac\{y^2\}\{9\}-\frac\{x^2\}\{16\}\=0$$

8. ¿Cuál de los siguientes vectores representa a una ecuación paramétrica de la elipse:

a. $$(4\cos(t),2\sin(t))$$

b. $$(2\cos(t),2\sin(t))$$

c. $$(4\sec(t),2\tan(t))$$

d. $$(4\cos(t),4\sin(t))$$

e. $$(4\tan(t),2\sec(t))$$

Page 72: Diseño Instruccional Matemáticas. “Trigonometría y ...conacyt.upemor.edu.mx/informe3/archivos/PropiedadInt...Diseño Instruccional Matemáticas.“Trigonometría y Geometría

72

9. ¿Qué ecuación describe la siguiente figura?:

a. $$r\=\frac\{10\}\{3+2 \cos \{\theta\}\}$$

b. $$r\=\frac\{10\}\{2+2 \cos \{\theta\}\}$$

c. $$r\=\frac\{10\}\{3+3 \cos \{\theta\}\}$$

d. $$r\=\frac\{-10\}\{3+2 \cos \{\theta\}\}$$

e. $$r\=\frac\{10\}\{3+2 \cos \{2\theta\}\}$$

10. ¿Cuál de las siguientes opciones contiene la ecuación de la recta que se muestra en la

figura?

a. $$y\=-2x-\frac\{1\}\{2\}$$

b. $$y\=x+\frac\{1\}\{2\}$$

c. $$y\=-\frac\{1\}\{2\}x+\frac\{1\}\{2\}$$

d. $$y\=\frac\{1\}\{2\}x+1$$

e. $$y\=\frac\{1\}\{2\}x-1$$

11. ¿Cuál de las siguientes opciones contiene la ecuación de la recta que se muestra en la

figura?

Page 73: Diseño Instruccional Matemáticas. “Trigonometría y ...conacyt.upemor.edu.mx/informe3/archivos/PropiedadInt...Diseño Instruccional Matemáticas.“Trigonometría y Geometría

73

a. $$y\=2x-1$$

b. $$y\=3x+1$$

c. $$y\=2x+1$$

d. $$y\=\frac\{1\}\{2\}x+1$$

e. $$y\=\frac\{1\}\{2\}x-1$$

12. ¿Cuánto vale la distancia del punto A=(0,1) a la recta y=x?

a. $$1$$

b. $$\sqrt\{2\}$$

c. $$1/\sqrt\{2\}$$

d. $$1/2$$

e. $$\sqrt\{3\}/2$$

13. Determinar la ecuación de la mediatriz (perpendicular en su punto medio) del segmento (-

2,1), (3,-5):

a. $$10x-12y-29\=0$$

b. $$10x+12y+29\=0$$

c. $$y-2\=\frac\{5\}\{6\}(x+\frac\{1\}\{2\})$$

d. $$10x+12y-29\=0$$

e. $$y+2\=\frac\{6\}\{5\}(x+\frac\{1\}\{2\})$$

14. Determinar la ecuación de la recta que pasa por el punto (-3,1) y es paralela a la recta

determinada por los dos puntos (0,-2) y (5,2):

a. $$4x-5y+17\=0$$

b. $$4x+5y-17\=0$$

c. $$4x-5y-17\=0$$

d. $$-4x-5y+17\=0$$

e. $$4x+5y+17\=0$$

15. Determinar la ecuación de la recta que pasa por el punto (4,-1) y tiene ángulo de

inclinación de 135^o:

a. $$x+y-3\=0$$

b. $$x+y+3\=0$$

c. $$-x+y-3\=0$$

d. $$x-y-3\=0$$

e. $$-x-y-3\=0$$

Page 74: Diseño Instruccional Matemáticas. “Trigonometría y ...conacyt.upemor.edu.mx/informe3/archivos/PropiedadInt...Diseño Instruccional Matemáticas.“Trigonometría y Geometría

74

16. El interior de una antena de TV por satélite es un disco con forma paraboloide (finito) de

diámetro de 12 pies y profundidad de 2 pies. Encuentra la distancia desde el centro del

disco hasta el foco.

a. $$4.5 pies$$

b. $$3 pies$$

c. $$3 metros$$

d. $$6 pies$$

e. $$2.5 pies$$

17. En una red se encuentran una araña y una cucaracha como se muestra en la figura. Cada

nudo de la red se separa de otro por 1 cm. ¿A qué distancia se encuentra la araña de la

cucaracha?

a. $$\sqrt[]\{85\}$$7

b. $$\sqrt[]\{70\}$$

c. $$\sqrt[]\{49\}$$

d. $$\sqrt[]\{13\}$$

e. $$\sqrt[]\{90\}$$

18. En un parque hay un pequeño lago artificial circular, el cual se describe por la ecuación

x^2+y^2=30m. Si un pato está en la coordenada (6m,4m) se puede afirmar que:

a. El pato está afuera del lago

b. El pato está adentro del lago

c. El pato está en el centro del lago

d. El pato está cerca del centro del lago

e. El pato está en la orilla del lago

19. La base de un auditorio es de forma elíptica, tiene 20 metros de longitud y 16 metros de

ancho. Si cae una aguja sobre un foco, ¿a qué distancia está un foco del otro foco?

a. está a $$12$$ metros

b. está a $$20$$ metros

c. está a $$16$$ metros

d. está a $$4$$ metros

e. está a $$6$$ metros

20. ¿Cuál es el valor exacto del \cos 30^\circ?

a. $$\frac\{1\}\{2\}$$

b. frac\{2\}\{\sqrt\{3\}\}$$

c. $$\frac\{\sqrt\{2\}\}\{2\}$$

d. $$\frac\{\sqrt\{3\}\}\{2\}$$

Page 75: Diseño Instruccional Matemáticas. “Trigonometría y ...conacyt.upemor.edu.mx/informe3/archivos/PropiedadInt...Diseño Instruccional Matemáticas.“Trigonometría y Geometría

75

e. $$\frac\{1\}\{\sqrt\{2\}\}$$

21. ¿Cuál es el valor exacto del \cos 45^\circ?

a. $$\frac\{1\}\{2\}$$

b. $$\frac\{2\}\{\sqrt\{3\}\}$$

c. $$\frac\{\sqrt\{2\}\}\{3\}$$

d. $$\frac\{\sqrt\{3\}\}\{2\}$$

e. $$\frac\{1\}\{\sqrt\{2\}\}$$

22. ¿Cuál es el valor exacto del \sin 30^\circ?

a. $$\frac\{1\}\{2\}$$

b. $$\frac\{2\}\{\sqrt\{3\}\}$$

c. $$\frac\{\sqrt\{2\}\}\{2\}$$

d. $$\frac\{\sqrt\{3\}\}\{2\}$$

e. $$\frac\{1\}\{\sqrt\{2\}\}$$

23. ¿Cuál es el valor exacto del \sin 60^\circ?

a. $$\frac\{1\}\{2\}$$

b. $$\frac\{2\}\{\sqrt\{3\}\}$$

c. $$\frac\{\sqrt\{2\}\}\{2\}$$

d. $$\frac\{\sqrt\{3\}\}\{2\}$$

e. $$\frac\{1\}\{\sqrt\{2\}\}$$

24. El ángulo de inclinación respecto a la horizontal del vector B=(-1,1) es:

a. $$60^\circ$$

b. $$90^\circ$$

c. $$135^\circ$$

d. $$180^\circ$$

e. $$0^\circ$$

25. El producto punto entre los vectores A=(2,-5) y B=(1,-2) vale:

a. $$A \cdot B\=12$$

b. $$A \cdot B\=(3,-7)$$

c. $$A \cdot B\=12$$

d. $$A \cdot B\=(2,10) $$

e. $$A \cdot B\=-8$$

26. El tamaño del vector C=(1,0) es:

a. $$|A|\=1$$

b. $$|A|\=2$$

c. $$|A|\=\sqrt\{2\}$$

d. $$|A|\=\sqrt\{4\}$$

e. $$|A|\=1/2$$

27. El tamaño del vector A=(1,1) es:

a. $$|A|\=1$$

b. $$|A|\=2$$

c. $$|A|\=\sqrt\{2\}$$

d. $$|A|\=\sqrt\{4\}$$

Page 76: Diseño Instruccional Matemáticas. “Trigonometría y ...conacyt.upemor.edu.mx/informe3/archivos/PropiedadInt...Diseño Instruccional Matemáticas.“Trigonometría y Geometría

76

e. $$|A|\=1/2$$

28. Encuentra la ecuación de la elipse con centro en el origen, foco F(3,0) y excentricidad

e=0.6.

a. $$\displaystyle\{\frac\{x^2\}\{25\}+\frac\{y^2\}\{16\}\=1\}$$

b. $$\displaystyle\{\frac\{x^2\}\{16\}+\frac\{y^2\}\{25\}\=1\}$$

c. $$\displaystyle\{\frac\{x^2\}\{25\}+\frac\{y^2\}\{9\}\=1\}$$

d. $$\displaystyle\{\frac\{x^2\}\{3\}+\frac\{y^2\}\{9\}\=1\}$$

29. Encuentra la excentricidad de la elipse cuya ecuación es 3x^2+2y^2-18=0.

e. $$\displaystyle\{e\=\frac\{1\}\{\sqrt\{3\}\}\}$$

f. $$\displaystyle\{e\=\frac\{1\}\{0.58\}\}$$

g. $$\displaystyle\{e\=\frac\{1\}\{\sqrt\{9\}\}\}$$

h. $$\displaystyle\{e\=\frac\{\sqrt\{3\}\}\{\sqrt\{9\}\}\}$$

30. Si se tienen los siguientes vectores A=(2,2), B=(1/2,1/2) y C=(1,3), ¿Cuál será el valor de

(A\cdot B)\cdot C?

i. $$(1,3)$$

j. $$(2,6)$$

k. $$8$$

l. $$4$$

m. $$(1,1)$$

31. Se tienen tres vectores en el plano, dados por: A=(1,1), B=(0,1) y C=(0,-2). El vector D=3 A-

B+\frac{1}{2}C es:

n. $$(1,0)$$

o. $$(1,4)$$

p. $$(3,1)$$

q. $$(3,-1)$$

r. $$(1,-3)$$

32. Se tiene el vector A=(1,-4). Si B es un vector perpendicular a A, entonces:

s. $$B\=(2,-8)$$

t. $$B\=(-4,1)$$

u. $$B\=(-1,4)$$

v. $$B\=(4,1)$$

w. $$B\=(1,1)$$

33. La suma de los vectores en el plano, dados por A=(-1,1), B=(0,3) y C=(2,-1) es:

x. $$(1,-3)$$

y. $$(3,-1)$$

z. $$(-1,3)$$

aa. $$(-1,-3)$$

bb. $$(1,3)$$

34. ¿Qué ángulo respecto al horizonte tendrá el sol cuando un objeto proyecta una sombra

del mismo largo que su altura?

cc. $$30^\circ$$

dd. $$45^\circ$$

Page 77: Diseño Instruccional Matemáticas. “Trigonometría y ...conacyt.upemor.edu.mx/informe3/archivos/PropiedadInt...Diseño Instruccional Matemáticas.“Trigonometría y Geometría

77

ee. $$60^\circ$$

ff. $$90^\circ$$

gg. $$0^\circ$$

35. En la figura, el área del triángulo ABC es 60. Si la medida de DB es la tercera parte de la

medida CB, entonces el área del triángulo ADB es:

hh. 20

ii. 30

jj. 40

kk. 45

ll. 50

36. En el triángulo de la figura, la razón \frac{3}{4} expresa el valor de:

mm. $$\sin u$$

nn. $$\cos u$$

oo. $$\tan u$$

pp. $$\cot u$$

qq. $$\sec u$$

37. El valor exacto del \cos(15^\circ) es:

rr. $$\frac\{1\}\{2\sqrt\{2\}\}$$

ss. $$\frac\{1+\sqrt\{3\}\}\{2\sqrt\{2\}\}$$

tt. $$\frac\{\sqrt\{3\}\}\{2\}$$

uu. $$\frac\{1+\sqrt\{2\}\}\{2\}$$

vv. $$\frac\{1\}\{2\}$$

38. Un albañil quiere dibujar una circunferencia en un terreno. Para ello, entierra un poste en

(0,0) y sujeta un lazo en el poste para hacer un radio hasta (2,4). ¿Cuál es la ecuación de la

circunferencia?

ww. $$x^2+y^2\=20$$

Page 78: Diseño Instruccional Matemáticas. “Trigonometría y ...conacyt.upemor.edu.mx/informe3/archivos/PropiedadInt...Diseño Instruccional Matemáticas.“Trigonometría y Geometría

78

xx. $$x^2+y^2\=8$$

yy. $$x^2+y^2\=16$$

zz. $$x^2+y^2\=24$$

aaa. $$x^2+y^2\=25$$

39. ¿Qué ecuación podría describir la superficie de una rampa vista transversalmente? La base

mide 2 metros y su altura 1 metro.

bbb. $$y\=\frac\{1\}\{2\}x$$

ccc. $$y\=2x$$

ddd. $$y\=x$$

eee. $$y\=2x+1$$

fff. $$y\=2x-1$$

40. La órbita que describe la Tierra alrededor del Sol es aproximadamente una elipse, con el

Sol en uno de sus focos. Si el eje mayor de la órbita elíptica es de 300000 km, y la

excentricidad es de 0.017 aproximadamente. Hallar la distancia máxima de la Tierra al Sol.

ggg. $$152550 km$$

hhh. $$3853493 km$$

iii. $$1534521 km$$

jjj. $$532558 km$$

kkk. $$987982558 m$$

41. La órbita que describe la Tierra alrededor del Sol es aproximadamente una elipse, con el

Sol en uno de sus focos. Si el eje mayor de la órbita elíptica es de 300000 km, y la

excentricidad es de 0.017 aproximadamente. Hallar la distancia mínima de la Tierra al Sol.

lll. $$147450 km$$

mmm. $$183431 m$$

nnn. $$980678 km$$

ooo. $$675890 m$$

ppp. $$89787 km$$

42. Un triángulo tiene sus vértices en los puntos A=(2,3), B=(6,9) y C=(8,1). ¿Cuál es la

coordenada su baricentro?

qqq. $$(4,6)$$

rrr. $$\left(\frac\{16\}\{3\},\frac\{13\}\{3\}\right)$$

sss. $$\left(\frac\{65\}\{11\},\frac\{52\}\{11\}\right)$$

ttt. $$\left(\frac\{13\}\{3\},\frac\{16\}\{3\}\right)$$

uuu. $$\left(\frac\{52\}\{11\},\frac\{65\}\{11\}\right)$$

43. Un triángulo tiene sus vértices en los puntos A=(0,0), B=(1,0) y C=(1/2,\sqrt{3}/2). ¿Cuánto

vale su perímetro?

vvv. 1

Page 79: Diseño Instruccional Matemáticas. “Trigonometría y ...conacyt.upemor.edu.mx/informe3/archivos/PropiedadInt...Diseño Instruccional Matemáticas.“Trigonometría y Geometría

79

www. $$1/2$$

xxx. $$3/4$$

yyy. 3

zzz. 4

44. ¿Cuál de las siguientes opciones contiene la ecuación que describe a una recta

perpendicular a otra, dada por la ecuación 4x-3y=6?

aaaa. $$4x+3y\=6$$

bbbb. $$4x-3y\=-6$$

cccc. $$3x-4y\=-6$$

dddd. $$3x-4y\=6$$

eeee. $$3x+4y\=6$$

45. ¿Cuál de las siguientes opciones contiene la ecuación que describe a una recta paralela a

otra dada por la ecuación 4x-3y=6?

ffff. $$4x+3y\=6$$

gggg. $$-4x-3y\=-6$$

hhhh. $$8x+6y\=-6$$

iiii. $$8x+6y\=6$$

jjjj. $$8x-6y\=6$$

46. Suponga que usted tiene un terreno plano de la siguiente forma y requiere pasar un tubo

de PVC por debajo del terreno que conecte el punto A con el punto B. ¿Cuál es la longitud

mínima que debe tener el tubo?:

kkkk. $$d_\{min\}\=\sqrt[]\{(7+5)^2+(12+9)^2\}$$

llll. $$d_\{min\}\=\sqrt[]\{(7-5)^2+(12-9)^2\}$$

mmmm. $$d_\{min\}\=\sqrt[]\{(7+5)^2+(-18+4)^2\}$$

nnnn. $$d_\{min\}\=\sqrt[]\{(7+5)^2+(18+4)^2\}$$

oooo. $$d_\{min\}\=\sqrt[]\{(7-5)^2+(18+4)^2\}$$

47. En un círculo de centro en el origen y radio igual a 5, determinar la forma normal de la

ecuación de su tangente en el punto (-3,4):

Page 80: Diseño Instruccional Matemáticas. “Trigonometría y ...conacyt.upemor.edu.mx/informe3/archivos/PropiedadInt...Diseño Instruccional Matemáticas.“Trigonometría y Geometría

80

pppp. $$\frac\{-3\}\{5\}x+\frac\{4\}\{5\}y-5\=0$$

qqqq. $$\frac\{3\}\{5\}x+\frac\{4\}\{5\}y-5\=0$$

rrrr. $$\frac\{-3\}\{5\}x+\frac\{-4\}\{5\}y-5\=0$$

ssss. $$\frac\{3\}\{5\}x+\frac\{4\}\{5\}y-5\=0$$

tttt. $$\frac\{3\}\{5\}x+\frac\{4\}\{5\}y+5\=0$$

48. ¿En qué punto se intersectan las siguientes rectas?: y=3x+2, y=-5x-2

uuuu. $$(0,0)$$

vvvv. $$(1,1)$$

wwww. $$(1/2,1/2)$$

xxxx. $$(-1/2,1/2)$$

yyyy. $$(-1,1/2)$$

49. Determinar los valores que deben tener los coeficientes de la ecuación general Ax+By+C=0

de una recta, para que pase por los dos puntos (-1,4) y (3,-2). Determinar la ecuación de la

recta:

zzzz. Los coeficientes son $$A\=3, B\=2, C\=-5$$ y la ecuación es $$3x+2y-

5\=0$$

aaaaa. Los coeficientes son $$A\=-3, B\=2, C\=-5$$ y la ecuación es $$-3x+2y-

5\=0$$

bbbbb. Los coeficientes son $$A\=2, B\=3, C\=-5$$ y la ecuación es $$2x+3y-

5\=0$$

ccccc. Los coeficientes son $$A\=3, B\=-2, C\=-5$$ y la ecuación es $$3x-2y-

5\=0$$

ddddd. Los coeficientes son $$A\=-3, B\=2, C\=5$$ y la ecuación es $$-

3x+2y+5\=0$$

50. Sea C una curva con ecuaciones paramétricas x=t^3-3t; y=t^2-5t-1, donde t está en R.

Encuentre una ecuación de la recta tangente a C en el punto correspondiente a t=2:

eeeee. $$y+7\=\frac\{-1\}\{9\}(x-2)$$ o su equivalente $$x+9y+61\=0$$

fffff. $$y+7\=\frac\{1\}\{9\}(x-2)$$ o su equivalente $$x+9y\=61$$

ggggg. $$y+7\=\frac\{+1\}\{9\}(x-2)$$

hhhhh. $$y-7\=\frac\{+1\}\{9\}(x-2)$$

iiiii. $$y+7\=\frac\{1\}\{9\}(x-2)$$

51. ¿Qué ecuación describe la siguiente figura (rosa de cuatro pétalos)?

Page 81: Diseño Instruccional Matemáticas. “Trigonometría y ...conacyt.upemor.edu.mx/informe3/archivos/PropiedadInt...Diseño Instruccional Matemáticas.“Trigonometría y Geometría

81

jjjjj. $$r\=a \sin\{20\}$$, donde $$a>0$$

kkkkk. $$r\=-2a \cos\{20\}$$, donde $$a>0$$

lllll. $$r\=-a \sin\{20\}$$, donde $$a>0$$

mmmmm. $$r\=2a \cos\{20\}$$, donde $$a>0$$

nnnnn. $$r\=\frac\{1\}\{2\}a \cos\{20\}$$, donde $$a>0$$

52. Una circunferencia representada por la ecuación (x-3)^2+(y+1)^2=9 tiene su centro en:

ooooo. $$(-3,1)$$

ppppp. $$(3,1)$$

qqqqq. $$(-1,3)$$

rrrrr. $$(3,-1)$$

sssss. $$(-3,-1)$$

53. Los vértices de la hipérbola representada por la ecuación \frac{x^2}{16}-\frac{y^2}{9}=1

son:

ttttt. $$(0,-4)$$ y $$(0,4)$$

uuuuu. $$(0,3)$$ y $$(0,-3)$$

vvvvv. $$(-16,0)$$ y $$(9,0)$$

wwwww. $$(-4,0)$$ y $$(4,0)$$

xxxxx. $$(-3,0)$$ y $$(4,0)$$

54. La relación x^2+y^2=9 representa:

yyyyy. Una parábola con vértice en el origen y foco en $$(3,0)$$.

zzzzz. Una circunferencia de radio $$3$$ y centro en el origen.

aaaaaa. Una circunferencia de radio $$9$$ y centro en el origen.

bbbbbb. Una elipse centrada en el origen con semieje mayor igual a $$9$$.

cccccc. Una parábola con vértice en el origen y foco en $$(0,3)$$.

55. La elipse de la figura siguiente se representa por la ecuación:

Page 82: Diseño Instruccional Matemáticas. “Trigonometría y ...conacyt.upemor.edu.mx/informe3/archivos/PropiedadInt...Diseño Instruccional Matemáticas.“Trigonometría y Geometría

82

dddddd. $$x^2 /4 + y^2/9 \= 1$$

eeeeee. $$x^2 /2 + y^2/3 \= 1$$

ffffff. $$x^2 /3 + y^2/2 \= 1$$

gggggg. $$3 x^2 + 2 y^2 \= 1$$

hhhhhh. $$x^2 /9 + y^2/4 \= 1$$

56. La ecuación que representa la siguiente gráfica es:

iiiiii. $$2x\=y^2+8y+22$$

jjjjjj. $$2x\=y^2-8y+22$$

kkkkkk. $$2x\=x^2+8x+22$$

llllll. $$x\=y^2+8y+22$$

mmmmmm. $$2x\=y^2-8y+22$$

57. La ecuación de una circunferencia con centro en el origen y diámetro igual a 8 se

representa por la ecuación:

nnnnnn. $$x^2+y^2\=8$$

oooooo. $$x^2+y^2\=\sqrt\{8\}$$

pppppp. $$x^2+y^2\=16$$

qqqqqq. $$x^2+y^2\=64$$

rrrrrr. $$x^2+y^2\=4$$

58. Determine una ecuación para la parábola con vértice en (4,-1), eje paralelo al eje y y que

pasa por el origen:

ssssss. $$(x-4)^2\=16(y+1)$$

tttttt. $$(x-4)^2\=16(y+1)^2$$

Page 83: Diseño Instruccional Matemáticas. “Trigonometría y ...conacyt.upemor.edu.mx/informe3/archivos/PropiedadInt...Diseño Instruccional Matemáticas.“Trigonometría y Geometría

83

uuuuuu. $$(x-4)^2\=16(y-1)$$

vvvvvv. $$(x-1)^2\=16(y+4)$$

wwwwww. $$(x-1)^2\=16(y-1)$$

Page 84: Diseño Instruccional Matemáticas. “Trigonometría y ...conacyt.upemor.edu.mx/informe3/archivos/PropiedadInt...Diseño Instruccional Matemáticas.“Trigonometría y Geometría

84

Ejercicios

Vectores

1. Se tiene un vector u=(3,2), ¿ Cuáles son las coordenadas al que apunta este vector si comienza

del punto A=(-1,2) ?

a. El punto final queda en (2,4).

b. El punto final queda en (4,4).

c. El punto final queda en (4,0).

2. ¿Cuáles son las coordenadas del punto final de un vector v=(3,-2) si tiene como punto inicial

A=(-1,2).

a. (3,-2)

b. (-2,-2)

c. (2,2)

3. Dos pares ordenados (a,b) y (c,d), ya sea que representen un punto o un vector son iguales

entre si sólo si cumplen que a=c y b=d. De esta manera encuentra que valores de x e y

cumplen que: (x-2y,2x+y)=(-1,3).

a. x=-1, y=-1

b. x=1, y=1

c. x=\frac{7}{5}, y=\frac{1}{5}

4. En la siguiente construcción se tiene un vector que va del punto Q al punto R. ¿A qué punto

llegaría este mismo vector si empezará del punto S?

a. (6,2)

b. (-1,-2)

c. (5,-4)

5. Se tiene un punto indeterminado P=(x,y), pero que cumple la condición de que el vector que

va del punto A=(-1,-2) a P es igual que el vector que va de B=(2,4) a C=(8,-2). Determina las

coordenadas de P y elige la opción correcta.

a. P=(7,-8)

b. P=(5,-8)

c. P=(5,-4)

Page 85: Diseño Instruccional Matemáticas. “Trigonometría y ...conacyt.upemor.edu.mx/informe3/archivos/PropiedadInt...Diseño Instruccional Matemáticas.“Trigonometría y Geometría

85

6. Se tienen los siguientes vectores: u=(0,1), v=(3,2) y w=(-1,4). De las opciones siguientes elige

un vector t, tal que verifique la relación: w+v=t-u

a. t=(2,7)

b. t=(2,5)

c. t=(-4,3)

7. En la siguiente construcción se tienen los puntos A, B , C y D. Si se trazan cuatro vectores entre

ellos, el primero del punto A al B, el segundo de B a C, el tercero de C a D y el último de D a A,

¿cuánto vale el vector suma de estos cuatro?

a. (0,0)

b. (0,1)

c. (-1,0)

8. Imagina que un avión se mueve desde su punto de despegue en línea recta recorriendo 300

Km hacia el este y 100 Km hacia el norte de su punto de partida. En ese punto vira y después

de cierto tiempo, se desplaza a partir de ese punto otros 100 Km al norte y 50 Km hacia el

oeste. ¿A qué distancia se encuentra el avión del punto de partida?

a. 320.16km

b. 403.11km

c. 206.155km

9. Una persona inicia un viaje en automóvil y llega a su destino situado a 100Km al sur. La

carretera que tomó pasa por una población situada a 30Km al este y 10Km al sur del punto de

partida. ¿Qué distancia, en línea recta, hay del poblado al destino final del viajero?

a. distancia=94.86 Km

b. distancia=114.02 Km

c. distancia=130.38 Km

10. Un triángulo tiene sus vértices en los puntos A=(1,2), B=(-1,2) y C=(0,-4). De entre las opciones

de abajo, escoge la que contenga tres vectores que representen los lados del triángulo y el

valor de su perímetro.

a. Vectores: (2,0), (-1,6) y (-1,-6), Perímetro: 14.1655.

b. Vectores: (0,4), (-1,-2) y (1,-2), Perímetro: 8.47214.

c. Vectores: (1,2), (-1,2) y (0,-4), Perímetro: 8.47214.

11. Tenemos dos vectores, u=(1,2) y v=(3,1), analicen la siguiente operación, si tomamos los

siguientes valores de t: t=-1, t=1 y t=\frac{1}{2}: w=u+ t v. Los valores de w correspondientes

son:

Page 86: Diseño Instruccional Matemáticas. “Trigonometría y ...conacyt.upemor.edu.mx/informe3/archivos/PropiedadInt...Diseño Instruccional Matemáticas.“Trigonometría y Geometría

86

a. w=(-2,1), w=(4,3), w=(\frac{5}{2},\frac{5}{2})

b. w=(2,-1), w=(-4,-3), w=(\frac{5}{2},\frac{5}{2})

c. w=(2,1), w=(4,-3), w=(-\frac{5}{2},\frac{5}{2})

12. ¿Por qué escalar t hay que multiplicar un vector u=(3,-2) para obtener el vector v=(-1,2/3)?

a. t=-\frac{1}{3}

b. t=3

c. No existe el valor porque no apuntan en la misma dirección.

13. Se tiene un vector u=(3,2) en la construcción que se muestra. Utilizando la barra deslizadora,

encuentra el valor de t necesario para que el tamaño del vector w=t u sea aproximadamente

1. Calcula el valor exacto de t utilizando las propiedades de los vectores. ¿Cuáles son los

valores de t encontrados?

a. Valor aproximado: t=0.27. Valor exacto: t=0.27

b. Valor aproximado: t=0.28. Valor exacto: t=\frac{1}{\sqrt{13}}

c. Valor aproximado: t=0.56. Valor exacto: t=\frac{2}{\sqrt{13}}

14. Tenemos un vector v=(4,2), de las opciones siguientes, escoge un vector w que sea unitario y

que apunte en la misma dirección de v.

a. w=(\frac{2}{\sqrt{5}},\frac{1}{\sqrt{5}})

b. w=(\frac{1}{\sqrt{5}},\frac{2}{\sqrt{5}})

c. w=(-\frac{1}{\sqrt{5}},-\frac{2}{\sqrt{5}})

15. Como se mencionó en las definiciones, un vector es perpendicular a otro si su producto punto

es igual a cero, pues esto indica que el ángulo entre ellos es de 90^\circ (o \pi/2 radianes). En

la siguiente construcción, encuentra un vector perpendicular a u=(1.5,1) usando los vectores

i=(1,0) y j=(0,1) y las operaciones vectoriales (producto punto, producto escalar y suma de

vectores). ¿Cuál de las siguientes expresiones es adecuada para esta relación y cuál es el valor

de un vector perpendicular a u?

Page 87: Diseño Instruccional Matemáticas. “Trigonometría y ...conacyt.upemor.edu.mx/informe3/archivos/PropiedadInt...Diseño Instruccional Matemáticas.“Trigonometría y Geometría

87

a. Expresión: (u \cdot j) i + (u \cdot i) j. Valor: (1,1.5).

b. Expresión: -(u \cdot j) i + (u \cdot i) j. Valor: (-1,1.5).

c. Expresión: (u \cdot i) i + (u \cdot j) j. Valor: (1.5,1).

Puntos y rectas

1. Si A(8,10) y B(4,2) son puntos extremos del Segmento dirigido AB. Hallar las coordenadas del

punto C(x,y) que divide a este segmento en la razón AC/CB=\frac{2}{3}.

a. C(6,6)

b. C(\frac{32}{5},\frac{34}{5})

c. C(\frac{64}{5},\frac{40}{5})

2. Si A(5,0) y B(-1,3) son puntos extremos del Segmento dirigido AB. ¿Cuáles son las coordenadas

del punto C(x,y) que divide a este segmento en la razón AC/CB=-4?

a. C(-3,4)

b. C(\frac{2}{-3},\frac{1}{-3})

c. C(1,2)

Page 88: Diseño Instruccional Matemáticas. “Trigonometría y ...conacyt.upemor.edu.mx/informe3/archivos/PropiedadInt...Diseño Instruccional Matemáticas.“Trigonometría y Geometría

88

Lugares geométricos I

1. Obtenga y seleccione la ecuación cartesiana del lugar geométrico de todos los puntos H=(x,y)

tales que la pendiente del segmento que conecta a cada punto de H con S=(1,1) es la mitad de

la pendiente del segmento que los conecta con T=(2,5).

a. y=\frac{1-3x}{x+3}

b. y=\frac{x+3}{x-3}

c. y=\frac{1+3x}{x-3}

2. Calcula y elige la ecuación que representa el lugar geométrico de los puntos cuyas distancias a

la recta definida por la ecuación cartesiana y=x y al punto F=(-1,1) son siempre iguales.

a. 2 + 2 x + \frac{x^2}{2} - 2 y + x y + \frac{y^2}{2}=0

b. 2 + 2 x - 2 y + 2 x y=0

c. 2 + 2 x + \frac{x^2}{2} - 2 y - x y + \frac{y^2}{2}=0

3. Obtenga una ecuación cartesiana del lugar geométrico de los puntos tales que el segmento

que va de cualquier punto de P=(x,y) a A=(-2,0) es perpendicular al segmento que va del punto

a B=(2,0).

a. x^{2}+ y^{2}=4

b. x + y=4

c. x^2 + y^2 =2

4. Una circunferencia pasa por los puntos A=(0,0), B=(7,0) y C=(3.5,3). Usando geogebra

determina la ecuación cartesiana de la circunferencia que pasa por ellos.

a. (x-3.5)^2+(y+0.54)^2=12.54

b. (x-3.5)^2+(y+0.04)^2=12.25

c. (x-0.04)^2+(y-3.5)^2=12.25

5. Determina y elige el centro y el radio de la circunferencia descrita por la ecuación cartesiana

x^2+y^2+4x-9y=0

a. Centro=(-2,4.5), Radio=4.92

b. Centro=(2,-4.5), Radio=24.25

c. Centro=(2,4.5), Radio=24.25

6. Elije la ecuación vectorial de la recta que es tangente en el punto A=(6,-4) a la circunferencia

cuya ecuación es: x^{2}+y^{2}-4x+2y-20=0

a. (x,y)=(6,-4)+t(3,4)

b. (x,y)=(6,-4)+t(-3,4)

c. (x,y)=(2,-1)+t(4,-3)

7. Determinación de los focos de la elipse. Se tiene una elipse descrita por la ecuación:

\begin{displaymath} \frac{x^2}{4}+\frac{y^2}{25}=1, \end{displaymath} por lo que su centro

está en (0,0) y sus semiejes valen a=2 y b=5. ¿Cuánto valen las coordenadas de los focos de la

elipse?

a. F1=(0,\sqrt{21}) y F2=(0,-\sqrt{21})

b. F1=(\sqrt{21},0) y F2=(-\sqrt{21},0)

c. F1=(\sqrt{25},0) y F2=(-\sqrt{25},0)

Page 89: Diseño Instruccional Matemáticas. “Trigonometría y ...conacyt.upemor.edu.mx/informe3/archivos/PropiedadInt...Diseño Instruccional Matemáticas.“Trigonometría y Geometría

89

8. El lado recto de una elipse es un segmento que se forma con los dos puntos de intersección de

la elipse con una recta perpendicular al eje mayor que pasa por un foco. Traducido a un dibujo

esta definición se muestra en la figura que sigue, donde los lados rectos se trazaron en color

rojo:

Es claro que toda elipse tiene dos lados rectos cuyo tamaño es igual. El problema dice así:

¿cuánto vale el tamaño del lado recto de una elipse descrita por la ecuación:

\begin{displaymath} \frac{x^2}{36}+\frac{y^2}{16}=1 \end{displaymath}?

a. 16/3

b. 8/3

c. 4/2

9. Escoge el valor del lado recto de una elipse que tiene focos en (2,2) y (5,2) y tiene un vértice

en (0,2).

a. 5.71

b. 2.85

c. 11.42

Lugares geométricos II

1. ¿Cuál es la ecuación de la parábola con vértice en el origen y foco F(2,0), y cuál es la ecuación

de su directriz?

a. Parábola: y^2=8x, Directriz: x=-2.

¿Cómo se hace?

Sabemos que:

\left|\left(x,y\right)-\left(2,0\right)\right|=\left|\left(x,y\right)-\left(-2,y\right)\right|,

\left|\left(x-2,y\right)\right|=\left|\left(x+2,0\right)\right|,

\sqrt{\left(x-2\right)^2+\left(y\right)^2}=\sqrt{\left(x+2\right)^2},

\left(x-2\right)^2+y^2}=\left(x+2\right)^2,

4-4x+x^2+y^2= 4+4x+x^2,

y^2=8 x.

Page 90: Diseño Instruccional Matemáticas. “Trigonometría y ...conacyt.upemor.edu.mx/informe3/archivos/PropiedadInt...Diseño Instruccional Matemáticas.“Trigonometría y Geometría

90

2. Calcule las coordenadas del foco, obtenga una ecuación cartesiana de la directriz y de la

parábola con vértice en el origen y que pasa por los puntos S(-5,5), T(-5,-5).

a. Parábola: y^2=-5x, Directriz: x=1.25

¿Cómo se hace?

Al graficar S y T nos damos cuenta que el eje de la parábola debe ser el eje x, y como la

parábola tiene vértice en el origen, entonces debe abrir hacia la izquierda. Por lo que

sabemos que su ecuación es de la forma y^2=4px. Por lo tanto la parábola tiene foco

F\left(\frac{-25}{20},0\right) y directriz x=\frac{25}{20}. De donde se deduce que su

ecuación es y^2=-5x.

3. Emplea la definición de la parábola para obtener una ecuación cartesiana de la parábola con

foco F(-2,-5) y directriz y=-4.

a. Parábola: x^2=-4x-2y-13.

¿Cómo se hace?

Un punto U(x,y) está sobre la parábola si:

|u-f|=|u-w|

|(x,y)-(-2,-5)|=|(x,y)-(x,-4)|,

|(x+2,y+5)|=|(x-x,y+4)|,

\sqrt{(x+2)^2+(y+5)^2} =\sqrt{(y+4)^2},

(x+2)^2+(y+5)^2}=(y+4)^2,

4+4x+x^2+25+10y+y^2=16+8y+y^2,

Page 91: Diseño Instruccional Matemáticas. “Trigonometría y ...conacyt.upemor.edu.mx/informe3/archivos/PropiedadInt...Diseño Instruccional Matemáticas.“Trigonometría y Geometría

91

x^2=-4x-2y-13.

4. Cuando se arroja una piedra desde un punto A, la piedra viaja aproximadamente a lo largo de

un arco parabólico con eje vertical. Si se arroja la piedra en una dirección que forma un ángulo

de 45^\circ con la horizontal, entonces el foco de la parábola está sobre una recta horizontal

que pasa por A.

Supongamos que una piedra que se lanza con este ángulo de elevación llega a una altura

máxima de 40m. ¿Qué distancia recorre la piedra horizontalmente hasta el momento de

alcanzar una altura igual a la del punto A?

a. 80m.

¿Cómo se hace?

Primero notemos que lo que deseamos es calcular la distancia entre los puntos A y B. Este

segmento que es perpendicular al eje focal de la parábola, cuyo punto medio es el foco y

cuyos extremos A y B, están sobre la parábola es el lado recto, y su longitud se llama ancho

focal y mide |4p|. De esta manera. La ecuación de la parábola que describe la piedra

lanzada es x^2=4py, donde p&lt;0. Entonces en y=p, x=\pm|2p|, por lo que la distancia

entre A y B, la distancia horizontal que recorre la piedra es de 160m.

5. Calcula la ecuación de la hipérbola con focos F1(0,5) y F2(0,-5); y con vértices V1(0,4) y V2(0,-

4).

a. \displaystyle{\frac{y^2}{16}-\frac{x^2}{9}=1}.

¿Cómo se hace?

Page 92: Diseño Instruccional Matemáticas. “Trigonometría y ...conacyt.upemor.edu.mx/informe3/archivos/PropiedadInt...Diseño Instruccional Matemáticas.“Trigonometría y Geometría

92

Observando la figura notamos que un punto U con coordenadas (x,y) está sobre la

hipérbola si cumple con:

\left\|U-F1\right\|-\left\|U-F2\right\|=2ª

\left\|\left(x,y\right)-\left(0,5)\right\|-\left\|\left(x,y\right)-\left(0,-5\right)\right\|=8

\left\|\left(x,y-5\right)\right\|-\left\|\left(x,y+5\right)\right\|=8

\sqrt{x^2+\left(y-5\right)^2}-\sqrt{x^2+\left(y+5\right)^2}=8

Con un poco de álgebra obtenemos que la ecuación es:

\displaystyle{\frac{y^2}{16}-\frac{x^2}{9}=1}

6. Obtén la ecuación de la hipérbola con centro en el origen y cuya longitud del eje conjugado es

2a=10, y del eje transversal es 2b=8, y con eje principal el eje x.

a. \displaystyle{\frac{x^2}{25}-\frac{y^2}{16}=1}.

¿Cómo se hace?

Puesto que la longitud del eje conjugado es 10, la parábola tiene su centro en el origen y

eje principal el eje x, las coordenadas de sus vértices son: (-5,0) y (5,0). Por otra parte

sabemos que la longitud del eje transversal es 8, por lo que b=4. Por tanto la ecuación es:

\frac{x^2}{25}-\frac{y^2}{16}=1. La hipérbola se ve como en la figura.

7. Calcula las coordenadas de los vértices y los focos de la hipérbola con ecuación \frac{x2}{25} -

\frac{y2}{9}=1.

a. De la ecuación deducimos que a=5 y b=3. Por lo que las coordenadas de los vértices

son (-5,0) y (5,0). Por otra parte, como c^2=b^2-a^2, entonces c^2=34, y las

coordenadas de los focos son (-\sqrt{34},0) y (\sqrt{34},0).

Page 93: Diseño Instruccional Matemáticas. “Trigonometría y ...conacyt.upemor.edu.mx/informe3/archivos/PropiedadInt...Diseño Instruccional Matemáticas.“Trigonometría y Geometría

93

8. Obtén la ecuación cartesiana de la hipérbola con centro en el origen, cuyos focos son F1(4,0) y

F2(-4,0), y que pasa por el punto S(14,24).

a. \displaystyle{\frac{x^2}{4}-\frac{y^2}{12}=1}.

¿Cómo se hace?

Sabemos que:

\left\|u-f1\right\|-\left\|u-f2\right\|=2ª

\left\|(14,24)-(4,0)\right\|-\left\|(14,24)-(-4,0)\right\|

\left\|(10,24)\right\|-\left\|(18,24)\right\|=4

Por lo tanto a=2. Por otra parte sabemos que: b^2=c^2-a^2, y de las coordenadas de los

focos sabemos que c=4. Por lo tanto b^2=c^2-a^2=16-4=12. De donde concluimos que la

ecuación de la hipérbola es: : \frac{x^2}{4}-\frac{y^2}{12}=1.

Coordenadas Polares

1. Se tiene un vector u=(1,-1), ¿Cuál es su representación en coordenadas polares si se usan

ángulos en radianes medidos de 0 a 2\pi?

a. u_\alpha=(\sqrt{2},\frac{7}{4} \pi)

b. u_\alpha=(2,\frac{1}{4} \pi)

c. u_\theta=(\sqrt{2},\frac{3}{4} \pi)

2. Se tiene un vector v=(-2,3), ¿Cuál es su representación en coordenadas polares si se usan

ángulos en grados medidos de 0^\circ a 360^\circ?

a. v_\alpha=(\sqrt{13},123.69^\circ)

b. v_\alpha=(13,303.69^\circ)

c. v_\alpha=(\sqrt{13},-123.69^\circ)

3. Utilizando las relaciones entre coordenadas polares y cartesianas, transforma la ecuación

y=x+1 a una ecuación polar de la forma R=f(\alpha) y elige la respuesta correcta.

a. R=\frac{1}{\sin \alpha-\cos\alpha}

b. R=\frac{1}{\sin \alpha +\cos\alpha}

c. R=-\frac{1}{\sin \alpha +\cos\alpha}

4. Transforme la ecuación polar R=\dfrac{1}{1-\cos\alpha} a una ecuación cartesiana y seleccione

la solución correcta de las opciones que se muestran.

a. y^{2}=2x+1

Page 94: Diseño Instruccional Matemáticas. “Trigonometría y ...conacyt.upemor.edu.mx/informe3/archivos/PropiedadInt...Diseño Instruccional Matemáticas.“Trigonometría y Geometría

94

b. y=2x^2+1

c. y^{2}=-2x-1

5. Si se tiene un vector en coordenadas polares descrito por u_\alpha=(3 + 3 \sin \alpha, \alpha),

de las opciones siguientes ¿qué gráfica se obtendrá si variamos \alpha entre 0 y 2 \pi

radianes?

a.

b.

c. 6. Siguiendo el razonamiento presentado para la elipse selecciona ahora la ecuación correcta en

coordenadas polares que describe a la hipérbola que tiene los focos en F1=(0,0) y F2=(4,0) y

tiene sus vértices en (1,0) y (3,0).

a. \begin{displaymath} R=\frac{3}{1+2 \cos \alpha } \end{displaymath}

b. \begin{displaymath} R=\frac{2}{1-3 \cos \alpha } \end{displaymath}

c. \begin{displaymath} R=\frac{3}{1-2 \cos \alpha } \end{displaymath}

7. Aplicando los razonamientos anteriores ahora determina y selecciona la ecuación polar de una

parábola con foco en F=(0,0) y vértice en V=(2,0).

a. \begin{displaymath} R=\frac{4}{1+\cos \alpha}. \end{displaymath}

b. \begin{displaymath} R=\frac{4}{1-\cos \alpha}. \end{displaymath}

c. \begin{displaymath} R=\frac{1}{1+4\cos \alpha}. \end{displaymath}

8. Finalmente, calcula y elige la ecuación polar de la circunferencia centrada en el origen y de

radio igual a l.

a. R=l

b. R=l \cos \alpha

c. R=l \sin \alpha

9. Se sabe que la trayectoria del cometa Halley tiene una excentricidad de e= 0.967990 y su

afelio vale 35.1 UA (unidades astronómicas), ¿Cuánto vale el lado recto de su trayectoria?

a. Lado recto=2.2471 UA.

b. Lado recto=69.0764 UA

Page 95: Diseño Instruccional Matemáticas. “Trigonometría y ...conacyt.upemor.edu.mx/informe3/archivos/PropiedadInt...Diseño Instruccional Matemáticas.“Trigonometría y Geometría

95

c. Lado recto= 1.12355 UA

10. Halla la excentricidad de la hipérbola que tiene como focos los puntos F1=(0, 0) y F2=(10, 0), y

6 como diferencia absoluta de las distancias a los focos.

a. excentricidad=\frac{5}{3}

b. excentricidad=\frac{10}{3}

c. excentricidad=\frac{6}{10}

11. ¿Cuál será la excentricidad de una elipse representada por la ecuación cartesiana:

\frac{x^2}{144}+\frac{y^2}{81}=1 ?

a. e=\frac{\sqrt{7}}{8}

b. e=\frac{12}{3\sqrt{7}}

c. e=\frac{3\sqrt{7}}{12}

12. Se tiene la ecuación cartesiana de una cónica: \frac{x^2}{9}-\frac{y^2}{4}=1. Selecciona una

ecuación polar de una cónica idéntica pero con un foco en el origen.

a. R=\frac{1.33}{1-1.2 \cos \alpha}

b. R=\frac{2.67}{1-0.36 \cos \alpha}

c. R=\frac{1.33}{1-2.67 \cos \alpha}

Ejercicios de Aplicación

1. El arco de un puente con una luz de 6m y una altura de 5m tiene una forma semielíptica. Un

camión de carga con una altura de 4m desea pasar por abajo. ¿Cuál es el ancho máximo

permitido para el camión?

a. 4.8 m

b. 3.6 m

c. 3.4 m

d. 4.9 m

2. Un jardinero quiere trazar una elipse que tenga de ancho 6m, ayudado con un lazo y dos

estacas. Las estacas las coloca en los focos separados entre sí 7m. ¿Cuál es la longitud del lazo?

a. 9.34 m

b. 13.36 m

c. 9.22 m

d. 9.64 m

3. La órbita del satélite Morelos alrededor de la Tierra tiene forma elíptica con respecto a ella en

uno de sus focos. Si la máxima y la mínima distancia del satélite a la Tierra es de 800 y 400

kilómetros, respectivamente, ¿Cuál es la excentricidad de la órbita?

a. 0.353

b. 0.234

c. 0.333

d. 0.345

4. El cable de suspensión de un puente colgante adquiere la forma de un arco de parábola. Los

pilares que lo soportan tienen una altura de 60m y están separados una distancia de 500m,

quedando el punto más bajo del cable a una altura de 10m sobre la calzada del puente.

Page 96: Diseño Instruccional Matemáticas. “Trigonometría y ...conacyt.upemor.edu.mx/informe3/archivos/PropiedadInt...Diseño Instruccional Matemáticas.“Trigonometría y Geometría

96

Tomando como eje x la horizontal que define el puente, y como eje y el de simetría de la

parábola, hallar la ecuación de esta. Calcular la altura de un punto situado a 80m del centro

del puente.

a. x^2-1250y+12500=0; 15.12m

b. x^2-125y+125=0; 15.12m

c. x^2-1250y+12500=0; 17m

d. x^2-125y+125=0; 17m

5. Se lanza una piedra horizontalmente desde la cima de una torre de 185m de altura con una

velocidad de 15m/s. Hallar la distancia del punto de caída al pie de la torre suponiendo que el

suelo es horizontal.

a. 92.5m

b. 95.2m

c. 90m

d. 98.5m

6. Un avión que vuela hacia el Sur a una altura de 1.500m y a una velocidad de 200km/h deja

caer una bomba. Calcular la distancia horizontal del punto de caída a la vertical del punto de

lanzamiento.

a. 972m

b. 927m

c. 729m

d. 792m

7. Un arco parabólico tiene una altura de 25m y una luz de 40m. Hallar la altura de los puntos del

arco situados 8m a ambos lados de su centro.

a. 21m

b. 12m

c. 25m

d. 18m

8. Un arco tiene forma de semielipse con una luz de 150m siendo su máxima altura de 45m.

Hallar la longitud de dos soportes verticales situados cada uno a igual distancia del extremo

del arco.

a. y=30\sqrt{2} metros.

b. y=1800 metros.

c. y=225\sqrt{8} metros.

d. y=25\sqrt{2} metros.

9. La tierra describe una trayectoria elíptica alrededor del Sol que se encuentra en un de los

focos. Sabiendo que el semieje mayor de la elipse vale 1485\times10^8km y que la

excentricidad es, aproximadamente, 1/62, hallar la máxima y la mínima distancias de la Tierra

al Sol.

a. Max: 1509\times10^8 km, Min: 1461\times10^8 km

b. Max: 1590\times10^8 km, Min: 1641\times10^8 km

c. Max: 2400\times10^8 km, Min: 1485\times10^8 km

d. Max: 1950\times10^8 km, Min: 1485\times10^8 km

Page 97: Diseño Instruccional Matemáticas. “Trigonometría y ...conacyt.upemor.edu.mx/informe3/archivos/PropiedadInt...Diseño Instruccional Matemáticas.“Trigonometría y Geometría

97

10. Se lanza un proyectil desde un punto A con una velocidad inicial de 1000m/s formando un

ángulo de 35° con la horizontal. Hallar el alcance del proyectil y la duración de la trayectoria.

a. 95800m, 118s.

b. 9580m, 181s.

c. 9058m, 118s.

d. 98050m, 181s.

11. Hallar el ángulo con el que se debe lanzar un proyectil a una velocidad de 400m/s para que su

alcance sea de 12000m. Hallar, asimismo, la duración de la trayectoria.

a. 23° 42’; 32.8s.

b. 23° 24’; 23.8s.

c. 24° 43’; 38.2s.

d. 32° 24’; 32.8s.

12. Se lanza un proyectil con un ángulo de elevación de 60° y una velocidad inicial de 800m/s.

Hallar el alcance y el vértice de la trayectoria.

a. 56500m, 25000m.

b. 565m, 250m.

c. 55600m, 23000m.

d. 556m, 23m.

Tangentes a cónicas

1. Usando geogebra, o por inspección, obtenga una ecuación de la tangente a la grafica de 3x^{2}

+ 4y^{2} + 2x - 3y=0 en el origen. Elija la respuesta correcta entre las opciones que se dan.

a. 2x-3y=0

b. x-3y=0

c. x-y=0

2. Aplica la idea del problema anterior para obtener la ecuación de la recta que es tangente en el

punto S=(2,-1) a la elipse cuya ecuación es x^{2}+4y^{2}=8.

a. x-2y-4=0

b. x - 2y=0

c. 2 x - y=0

3. Se tiene una elipse descrita por la ecuación 4x^2+2y^2=1 y un punto R=(1,1) fuera de ella.

Existen dos rectas tangentes a la elipse que pasan por R. Calcula, usando geogebra las

coordenadas de los dos puntos donde cada tangente toca a la elipse y elige la respuesta

correcta.

a. los puntos son: (-0.1,0.69) y (0.43,-0.36)

b. los puntos son: (0.1,0.7) y (0.4,0.4)

c. los puntos son: (-0.1,0.7) y (0.4,-0.3)

4. Dos rectas que pasan por le punto R=(-3,4) son tangentes a la parábola cuya ecuación es

y^{2}=16x. Calcula y elige las ecuaciones correctas de estas rectas.

a. 2x-3y+18=0 y 2x+y+2=0

b. 3x-2y+18=0 y x+2y+2=0

Page 98: Diseño Instruccional Matemáticas. “Trigonometría y ...conacyt.upemor.edu.mx/informe3/archivos/PropiedadInt...Diseño Instruccional Matemáticas.“Trigonometría y Geometría

98

c. 3x-2y-18=0 y x+2y-2=0

5. La ecuación de la tangente a la curva 4x^2+6y^2-24=0 en el punto A=(0,2) tiene la forma

y=mx+2. Si se resuelven simultáneamente ambas ecuaciones ¿cuál es el valor del

discriminante de las soluciones en x?

a. El discriminante es: 576m^2

b. El discriminante es: 576m^2-4(4+6m^2).

c. El discriminante es: 24m.

6. Obtener la ecuación de la tangente en el punto S=(3,4) a la hipérbola cuya ecuación es x^{2}-

\frac{y^{2}}{2}=1, y elegir entre las opciones la correcta.

a. 3x-2y-1=0

b. 3x-2y=0

c. 3y-2x-1=0

7. Usando geogebra determina la ecuación de la tangente a la cónica 3x^2 - 9x y - y^2 + x - y = 2

en el punto A=(-1,0) y selecciónala entre las opciones que se dan.

a. -5x+8y=5

b. -2x+4y=2

c. -2.5y+4x=2.5

8. Utilizando el método diferencial encuentra la pendiente m de la tangente a la ecuación 6x^2-

2x+6y+12=0 en un punto arbitrario (x_0,y_0) sobre ella.

a. m=-\frac{6x_0-1}{3}

b. m=-\frac{3}{6x_0-2}

c. m=\frac{3}{12x_0-2}

9. Se tiene una cónica representada por la ecuación x^2 + x y + y^2 + 2x + 4y+1 =0. También se

sabe que el punto A=(0,-2+\sqrt{3}) está en la cónica. De entre las opciones siguientes escoge

la que contenga la información correcta.

a. La cónica es una elipse y la pendiente de su tangente en A vale: m=-\frac{1}{2}

b. La cónica es una elipse y la pendiente de su tangente en A vale: m=-0.5029

c. La cónica es una hipérbola y la pendiente de su tangente en A vale: m=2

10. Dos cónicas con ecuaciones y^2=4x+3 y \frac{x^2}{9}+\frac{y^2}{2}=1 se intersectan. El ángulo

de intersección entre ambas curvas se define como el ángulo que forman las dos rectas

tangentes a ellas en el punto de intersección. Usando GeoGebra encuentra el ángulo de

intersección entre ellas y selecciona la respuesta correcta.

a. 127.46^\circ

b. 197.46^\circ

c. 297.46^\circ